Sie sind auf Seite 1von 70

Practice Test 2 Answers & Explanations

For Classes Starting 11/17/06 and Later

SAT

*SAT is a registered trademark of the College Entrance Examination Board, which was not involved in the production of, and does not endorse, this product.

Copyright 2007 Kaplan, Inc. All rights reserved. No part of this book may be reproduced in any form, by photostat, microfilm, xerography, or any other means, or incorporated into any information retrieval system, electronic or mechanical, without the written permission of Kaplan, Inc.

Section 1 (WritingEssay)
Grade 6 Essay
Knowledge and action are often presented as opposites: knowledge is passive and action is aggressive. This false dichotomy incorrectly calls the route of action thoughtless and the route of knowledge inactive. It is not more important to acquire knowledge or to take action; instead, it is essential to take action based on knowledge and logic. Acting without knowledge can be disastrous, as I have learned from personal experience. A year ago, I broke up with a boyfriend based on a vicious rumor about him that I believed. If I had waited to act and gained more knowledge, I would have learned that the rumor was completely untrue. In this case, the impulse to act before knowing all of the facts cost me a relationship that I had valued and enjoyed. I had become, in Heinrich Heines words, an unwitting agent of others who had manipulated me. Similarly, knowledge without action is ineffective. My oldest sister is an excellent example of this. She is a smart young woman, and she loved the masters degree program she is enrolled in. However, she has been unable to complete her degree because she hasnt acted. Shes acquired all of the knowledge that she needs, but as of yet she has not overcome her own inertia and finished the masters thesis that is required for graduation. Mere knowledge and passing tests is not enough; to meet her goals she must take the action of completing research and formulating her own theories. Some of the best examples of the need for a combination of knowledge and action can be found in the scientific and engineering communities. Cures and treatments for diseases are researched and tested for years before they are made available to the public. Medical scientists must have a basic amount of knowledge about the effectiveness and likely side effects of a treatment before they allow it to be prescribed to patients. In the same way, automobile makers spend years and millions of dollars testing designs to ensure safety, performance, and efficiency before new models are sold to the public. Knowledge cannot be separated from action, nor can action be separated from knowledge. What is best is to take action based upon knowledge. Graders Comments: The writer fully addresses the assignment, answering the question and effectively referring to the prompt throughout the essay, indicating that she took the time to read and understand what is being asked. She keeps a clear focus on the issues of action and knowledge. The introduction effectively presents the writers view on the issue, and each body paragraph presents a separate and clear point in support of the writers position. In addition, each body paragraph contains clear, specific, and relevant examples. The conclusion, though brief, neatly summarizes the writers opinion and ties it into the prompt. This structure and organization indicates that this writer took the time to plan her essay before moving on to the produce step. The writer also uses language successfully. Sentence structure is effectively varied throughout the essay. The writers vocabulary is advanced (dichotomy, inertia). In addition, the essay contains no significant errors; the writer clearly saved time to proofread her work.

SAT

Practice Test 2
1

section one

Grade 4 Essay
Theres a reason that the saying look before you leap has been around for so long and is so well known. Thats because it is so important to think things through and get information before you act on something. Things dont turn out so great for you if you dont have knowledge. Its better to be known as a person of thought than as a person of action. Its important to base decision and actions on information. Last year, an older friend of mine selected a college based just on what she thought of the college visit. She like the dorms and the people she met so thats where she applied and enrolled. But she found out during her first few weeks that the classes were too large and that a major she was interested in wasnt offered. If she had took the time to research the school and become knowledgeable about it. She could have prevented her mistake. Its like she bought into the advertising without knowing the product she was being sold. I wont let the same mistake happen to me. As Ive searched for the perfect college, Ive been looking at all kinds of information from websites and guidebooks to student surveys and college visits. Our government recognizes the value of knowledge before action. In the past few years, weve spend more and more money on the CIA and FBI and Homeland Security mostly to gather information. This information can help us protect the country and prevent another war. Also the government sees the value of knowledge before action in the court system. The victim of a crime isnt allowed to act out based on emotion or impulse. The court system provides a fair trial for everyone, with lawyers on both sides. A jury of 12 people must evaluate the information and come to a conclusion. Jury members usually take their duty to make an informed decision about the case very seriously. In conclusion, decisions and actions shouldnt be taken until a person has enough knowledge to really know what theyre getting into. When you plan for your future you need to base those plans on knowledge. The government also shows that it is not wise to act without knowledge. Acquiring useful knowledge is how we prevent ourselves from making mistakes. Graders Comments: This essay addresses the assignment and remains generally focused on the prompt. The exception is the second body paragraph, which is more a discussion of the writers friends college search than an example or reason supporting the writers point of view. The essay has a clear organization, indicating that the writer took time to Plan before producing, but there are few transitions between ideas, which makes her argument difficult to follow in places. The writer presents multiple examples to support her point of view. While the writing is competent, there is little variety in sentence structure or word choice. Several errors appear, including sentence fragments and incorrect verb tenses; had she spent more time on the proofread step, she might have caught them. However, these errors do not affect the readers understanding of the essay.

SAT

Practice Test 2
2

section one

Grade 2 Essay
Its alot more important to take action then acquiring knowledge. Life isnt just about acquiring knowledge. If you never take action then youre just sitting around. And what good is knowing stuff if you dont do anything about it? Thats how I always feel in my history class. We have to read all of these pages in our textbook. And listen to a lecture almost every day or watch some film about historical event. Then tests are all about the facts of what happened. Im not sure why any of this is important. Its like all this useless knowledge in school. But our teacher says people are supposed to learn from the past and not make the same mistakes. How does me learning all of these dates and names help whats going on today? The teachers never explained that. School sure isnt helping me. Willing to take action is more important then continuing to learn things. Maybe if more teachers talked about what we can do with the stuff were learning in school well then Id be more interested in classes. Graders Comments: This essay addresses the assignment, indicating that the writer read and understood the prompt. The essay has a basic organization, with the first paragraph introducing the topic, the second paragraph somewhat developing the topic, and the final paragraph restating the writers point of view. However, the writer does not maintain focus on the specific issue, moving instead to a discussion of his dissatisfaction with what is taught in school. Formulating a plan before beginning to write would have helped this writer keep his argument on track. The essay is particularly weak in terms of detail and development. The body paragraph uses examples from the writers experience in school, but these examples are not fully developed or connected to the topic. The writers choice of words is relatively limited, and some words, such as like and stuff, are too slangy for a formal essay. In addition, the essay contains several errors in sentence structure and usage, including alot instead of a lot, then instead of than, a parallelism issue in paragraph sentence 1, a missing apostrophe in whats, and numerous sentence fragments, including sentence 1 of paragraph 3.

SAT

Practice Test 2
3

section one

Section 2 (Math)
1. C Category: Equations Difficulty: Low Strategic Advice: Keep your numbers and number placements straight when multiplying and dividing. Getting to the Answer: This is one of the rare questions in which a calculator will come in handy. Large numbers like these can be difficult to divide by hand. 50,525 x 50,525 2,021 25 25,250 x 2. B 2,021 x x 25,250 25 1,010

section two

Category: Equations Difficulty: Low Strategic Advice: Translate from English into math step by step. Getting to the Answer: After 13 marbles are removed, 40 13 = 27 marbles are 1 27 = 9 marbles. After 9 marbles left. One-third of the remaining marbles equals 3 are removed, 27 9 = 18 marbles are left. 18 marbles out of the original 40 marbles remain in the jar. 18 9 = 40 20 Category: Number Properties Difficulty: Medium Strategic Advice: This is an excellent question for Picking Numbers, since there are variables in the answer choices. Getting to the Answer: Pick c = 2 and d = 3: (A) 3(2) + (3) = 6 + 3 = 9, which is odd. (B) (3) + 4 = 7, also odd. (C) 7(2 + 3) = 7(5) = 35, also odd. (D) (2)(3) + 1 = 6 + 1 = 7, also odd. (E) (2) + 4(3) = 2 + 12 = 14, the only even integer among the answer choices. (E) is correct. Category: Inequalities Difficulty: Medium Strategic Advice: The key here is to set up an inequality by accurately translating English into math. Getting to the Answer: The total time Dave spends doing his homework is the time he spends answering questions (7 minutes for each of 4 questions) plus the time he spends reading the chapter (t). Therefore, the total time equals 4 7 + t minutes. Since the total amount of time cannot exceed 45 minutes, 4 7 + t 45.

3.

4.

SAT

Practice Test 2
4

5.

SAT Category: Triangles Practice Test 2 Difficulty: Low 5 Strategic Advice: When you see an isosceles triangle, mark angles opposite equal sides as equal. Getting to the Answer: Since AC = BC, the angles opposite those sides are also equal, and a = b. Angle d is supplementary to angle b, so d = 180 b. Angle f is supplementary to angle a, so f = 180 a. Since a and b are equal, d and f are also equal. (D) is correct. You could eliminate (B) and (E) right away by eyeballing the diagram: angles a and c are obviously acute, while angles e and f are obviously obtuse, so (B) and (E) must be wrong.

6.

Category: Linear Graphs Difficulty: Medium Strategic Advice: Since this question involves Roman numerals, first consider the figure that appears most often in the answer choices. Eliminate answer choices after you evaluate each Roman numeral. Getting to the Answer: Start with Figure III. It shows a straight line that passes through the origin. The equation of this line is y = kx, where k is a constant, so y and x are directly proportional. As one variable increases in value, the other must increase as well. III has to be a part of the correct answer, so eliminate (A) and (B). Next, consider Figure I. It shows the graph of y = |kx|. The ratio of y to x is not constant, so x and y are not proportional. Choices containing I, (D) and (E), can also be eliminated. You dont even have to look at II to know that (C) is correct. Category: Non-Linear Graphs Difficulty: Medium Strategic Advice: When a function is graphed, x is on the x-axis and f(x) is on the y-axis. Getting to the Answer: Draw a line at y = 1 and count the number of times it intersects the graph of f(x) between x = 4 and x = 4.
y

7.

(4, 0)

(4, 0) y = 1 (0, 2)

It intersects the graph twice, so there are two solutions to the equation f(x) = 1. The correct answer is (C). 8. D Category: Averages Difficulty: High Strategic Advice: The average formula is average =

sum of terms . numbers of terms

Even if youre not sure what to do when you first look at an Averages problem, plugging the information you have into the formula will often make the next step

section two

clear. Getting to the Answer: Start by simplifying c and d, then find their average. c=

a + (a + 1) + (a + 2) 3a + 3 = =a+1 3 3 b + (b + 1) + (b + 2) 3b + 3 = =b+1 3 3

section two

d=

The average of c and d is:

(a + 1) + (b + 1) a + b + 2 a + b 2 a + b c+d = = = + = +1 2 2 2 2 2 2
If you have trouble keeping track of the variables, try Picking Numbers. Say a = 2 and b = 4. Then: c=

2 + (2 + 1) + (2 + 2) 9 = =3 3 3 4 + (4 + 1) + (4 + 2) 15 =5 d= = 3 3
The average of c and d is 3+5 8 = = 4. Plug a = 2 and b = 4 into each 2 2

answer choice to see which one equals 4. (A) 2 + 4 = 6 (B) 1 (C) (D) 2+4 6 = =3 2 2 2+4 6 +1= +1=3+1=4 2 2

(E) (2)(4) = 8 Only (D) is 4 when a = 2 and b = 4, so it must be correct. 9. 3/4 or .75 Category: Ratios Difficulty: Low Strategic Advice: Always do the same thing to both sides of an equation. Getting to the Answer: When you see a proportion, cross-multiply. 1 4 = x 3 (1)(3) = (4)(x) 3 = 4x 3 =x 4 3 x = = 0.75 4

SAT

Practice Test 2
6

10.

Category: Sequences Difficulty: Medium Strategic Advice: When a question asks for a large term, dont waste time writing out the entire sequence. Look for a pattern instead. Getting to the Answer: Since a group of four numbers is repeated to form this sequence, every 4th term is the same. Divide 42 by 4. The remainder is 2, so the 42nd term of the sequence is the same as the 2nd term, which is 1.

SAT

Practice Test 2
7

11.

40 Category: Angles Difficulty: Low Strategic Advice: Whenever you see intersecting lines, look for vertical or supplementary angles. Getting to the Answer: All you need to know here is that vertical angles are equal. Since EF bisects COD, it also bisects AOB; AOE = EOB = 20. AOB = AOE + EOB = 20 + 20 = 40 Since COD and AOB are vertical, they are equal. COD = AOB = 40 2 Category: Equations Difficulty: Medium Strategic Advice: When you have a system of equations, use substitution or combination to solve for the unknowns. Getting to the Answer: Since d is in an equation with a and c, you want to find a in terms of c or c in terms of a. Use the first two equations: a = 10b c b= 5 c a = 10 5 a = 2c Now plug this into the third equation and solve for d: a = dc 2c = dc 2=d

12.

13.

(2003 1988) 15 = =3 5 5 For each of the 3 five-year periods, you would multiply the number of CDs in Sonias collection by 2, so from 1988 to 2003, you would multiply by 2 2 2 = 23. Say x is the number of CDs in Sonias collection in 1988: x(23) = 600 x= 600 600 = = 75 23 8

section two

75 Category: Sequences Difficulty: Medium Strategic Advice: You can either set up an equation by translating from English into math, or you can work backwards to get to the correct answer choice. Getting to the Answer: To set up an equation, first calculate the number of fiveyear periods that passed between 1988 and 2003:

If you work backwards, set up a chart to help you keep track of the information. Since the collection doubles in size every five years, when you move backwards five years it will be halved. Year 2003 1998 1993 1988 14. Number of CDs 600 300 150 75

section two

1125 Category: Data Analysis Difficulty: Medium Strategic Advice: The whole pie chart always represents the total number. Getting to the Answer: As you can see from the chart, 20 percent of the trees are less than 3 years old and 25 percent of the trees are between 3 and 5 years old. Thus, 20 + 25 = 45 percent of the trees are 5 years old or younger. The number of trees that are 5 years old or younger is 45% 2,500 = 0.45 2,500 = 1,125. 40 Category: Complex Figures Difficulty: High Strategic Advice: When you dont think you have enough information to answer a Grid-In question, try thinking creatively about other ways to interpret the information you do have. Getting to the Answer: Although you know the perimeter of the rectangle, you dont know the lengths of its individual sides, so you dont know the lengths of the sides of the triangles. However, you dont need that information to solve the question. To find the perimeter of the figure, try grouping the sides of the triangles into groups of four, combining two sides of a smaller triangle with two sides of a larger triangle. Because the triangles are equilateral, you know that this is equivalent to grouping two lengths of the rectangle with two widths of the rectangle, which gives you the perimeter. You can make two such groupings with the outside edges of the triangles, so the total perimeter of the figure is twice the perimeter of the rectangle, or 40. Algebraically, call the long side of the rectangle x and the short side y. The perimeter of the rectangle is 2x + 2y = 20. The perimeter of the entire figure is four long sides (from the two larger triangles) and four short sides (from the smaller triangles), or 4x + 4y. 4x + 4y = 2(2x + 2y) = 2(20) = 40 110 Category: Equations Difficulty: High Strategic Advice: Set up a system of equations by translating carefully from English to math. Keep an eye on the units. Getting to the Answer: Let a be the basic enrollment fee in dollars and b be the additional charge in dollars for every 3 months. The total cost of the 9-month membership is a + 9 b = a + 3b = 60 and the total cost of the 3 15 15-month membership is a + b = a + 5b = 80. Combination will let you 3

15.

16.

SAT

Practice Test 2
8

solve for one of the variables quickly. Subtract the first equation from the second one and solve for b: a + 5b = 80 (a + 3b = 60) 2b = 20 20 = 10 b= 2 Use either equation to find a: a + 5b = 80 a + 5(10) = 80 a + 50 = 80 a = 30 The total cost of a 2-year, or 24-month, membership is: 24 a+ b = a + 8b = 30 + 8(10) = 30 + 80 = 110 3 17. 94 Category: Quadratic Equations Difficulty: Medium Strategic Advice: Only identical polynomials can equal each other for all values of x. Getting to the Answer: First, simplify the equation by multiplying through: 1 3 1 1 x + x+ = px5 + qx3 + rx + t 10 20 40 1 3 1 1 80x5 + 80 x + 80 x + 80 = px5 + qx3 + rx + t 10 20 40 80 x5 + 80x5 + 8x3 + 4x + 2 = px5 + qx3 + rx + t In order for this equation to be true for ALL values of x, the coefficients on the right side have to be equal to those on the left side: p = 80, q = 8, r = 4, t = 2. p + q + r + t = 80 + 8 + 4 + 2 = 94 You could also make the problem more concrete by Picking Numbers. Say x = 1. 1 3 1 1 x + x+ 10 20 40 1 1 1 80 (1)5 + (1)3 + (1) + 10 20 40 1 1 1 80 1 + + + 10 20 40 40 4 2 1 80 + + + 40 40 40 40 47 80 40 80 x5 + = px5 + qx3 + rx + t

SAT

Practice Test 2
9

] = p(1)5 + q(1)3 + r(1) + t


= p(1) + q(1) + r(1) + t =p+q+r+t =p+q+r+t

94 = p + q + r + t Since the equation is true for all values of x, you could plug in any number for x and get the same result.

section two

18.

Category: Inequalities Difficulty: High Strategic Advice: Make sure you work methodically and carefully, remembering to include all of the information in the question stem. Getting to the Answer: a2 b2 (a b)2 < 4(a b) a2 b2 (a2 2ab + b2) < 4(a b) a2 b2 a2 + 2ab b2 < 4(a b) 2ab 2b2 < 4(a b) 2b(a b) < 4(a b) b(a b) < 2(a b) b>2 Because a < b, (a b) < 0, so you can divide both sides by it, but you have to change the direction of the inequality. Because b is an integer, the least possible value for b is 3.

SAT

Practice Test 2
10

section two

Section 3 (Writing)
1. D Category: Pronouns Difficulty: Medium Issues: wordiness, verb tenses Strategic Advice: Use who only to refer to people. Getting to the Answer: As written, the sentence uses the incorrect relative pronoun who to refer to events. (D) eliminates the incorrect pronoun. (B) and (C) create grammatically incorrect sentences; additionally, (C) introduces an inconsistent verb tense and adds also, which is redundant with as well as. (E) uses the word too, which is redundant with as well as. Category: Parallelism Difficulty: Medium Issues: run-ons and fragments, style problems Strategic Advice: Verbs in compounds must be parallel in form. Getting to the Answer: Here, the compound verb areyet relying does not follow the rules of parallel structure. (C) makes the second verb parallel to the first. (B) and (E) create sentence fragments. (D) is grammatically incorrect because the additional subject pronoun it is unnecessary with the subject the common housecat. Category: Wordiness/Parallelism Difficulty: High Issues: subjectverb, ambiguity, transitions Strategic Advice: Verbs in a compound should be parallel in form. Getting to the Answer: Here, the verb phrases would be paid and to give are not parallel; additionally, the sentence is wordier than it needs to be. (C) is concise and also makes the verb forms parallel. The verb in (B) does not agree with its plural subject, concessions. (D) uses the pronoun them with no clear antecedent. (E) uses they without an antecedent and also changes the meaning of the sentence; the original tells us that both concessions were made, making ifor inappropriate. Category: SubjectVerb Difficulty: Medium Issues: wordiness, run-ons and fragments Strategic Advice: Dont be fooled by intervening phrases; make sure that you know which subject corresponds to which verb and that they agree. Getting to the Answer: Even though the singular tone appears closer to it in the sentence, the subject of the verb is is actually the plural hammers. (E) is the only choice that corrects this error without introducing new problems. (B) is unnecessarily wordy. (C) creates a sentence fragment. (D) does not address the error. Category: Modifiers/SubjectVerb Difficulty: High Issues: run-ons and fragments Strategic Advice: The test maker will frequently put a plural object directly in front

SAT

Practice Test 2
11

2.

3.

4.

5.

section three

section three

of a verb with a singular subjectmake sure all verbs agree with the appropriate nouns. Getting to the Answer: This sentence has two problems: the plural verb are does not agree with its singular subject the number, and the introductory phrase modifies number rather than people. (D) corrects both errors. (B) and (C) do not address the modifier problem. (E) creates a sentence fragment. 6. C Category: Ambiguity Difficulty: Medium Issues: modifiers Strategic Advice: A pronoun without a clear antecedent will be incorrect on the SAT. Getting to the Answer: As written, this sentence provides no clear antecedent for the pronoun it. Only (C) eliminates the ambiguous pronoun. (B), (D), and (E) do not address the error; additionally, (D) changes the meaning of the original sentence, and (E) uses an adjective to modify a verb. Category: Wordiness Difficulty: Low Issues: ambiguity Strategic Advice: Watch out for overly wordy structures that dont add meaning to the sentence. Getting to the Answer: As written, this sentence is unnecessarily wordy. (D) makes the same point concisely, without changing the meaning of the sentence. (B) is still unnecessarily wordy. (C) changes the meaning of the original sentence, which concerns when writers gather their information, not why. (E) uses the pronoun it without a clear antecedent. Category: Run-ons and Fragments/Parallelism Difficulty: Medium Issues: verb tenses, idioms Strategic Advice: The ing verb form can never be a predicate verb. Getting to the Answer: Despite this sentences length, it is a fragment because it lacks a predicate verb. (C) replaces forcing with forced, creating a complete, correct sentence; it also makes the verbs to rely and to use parallel. (B) introduces an inconsistent verb tense and fails to properly complete the idiom not onlybut also. (D) also incorrectly uses not only without but also. (E) does not address the parallelism error. Category: Passives/Modifiers Difficulty: High Issues: ambiguity Strategic Advice: An introductory modifying phrase generally modifies the first noun or pronoun that follows it. Getting to the Answer: The passive voice here results in a modifier error; concerned college students, not rainforest ecology, are guided by Dr. Peck. Only (B) makes the correction. In (C), the opening phrase modifies the daily testing and water analysis.

7.

8.

9.

SAT

Practice Test 2
12

(D) uses the pronoun it without a clear antecedent. In (E), the opening phrase modifies Floridas rainforest ecology. 10. C Category: Comparisons Difficulty: Medium Issues: wordiness Strategic Advice: Whenever you come across a comparison, make sure that logical things are being compared. Getting to the Answer: This sentence makes an illogical comparison between the wakes of canoes and motorboats. (B) and (C) both correct this error, but (B) is unnecessarily wordy. Neither (D) nor (E) addresses the error. Category: Idioms Difficulty: Low Issues: wordiness Strategic Advice: Most tested idioms will hinge on preposition usage, but some may also concern the proper verb form in context. Getting to the Answer: The -ing verb form thinking is not correct with because of. (C) corrects the error. (B) does not address the error. (D) and (E) are unnecessarily wordy and (D) charges the meaning of the sentence. Category: Correct as Written Difficulty: High Issues: subjectverb, verb tenses, idioms Strategic Advice: A total of between five and eight Writing section questions will be correct as written. Getting to the Answer: This sentence is correct as written. (A) agrees with its plural subject and uses an appropriate verb tense in context. (B), (C), and (D) are idiomatically correct. Category: Pronouns Difficulty: Medium Issues: subjectverb, idioms Strategic Advice: Read sentences like this one with just the underlined pronoun. Since you wouldnt say him was awarded, the pronoun should be in the subjective case. Getting to the Answer: The pronoun him in (A) is part of the subject of a clause, so it should be the subjective case he. The verb phrase in (B) agrees with its compound subject. (C) and (D) are idiomatically correct. Category: Idioms Difficulty: High Issues: subjectverb Strategic Advice: Most Idioms questions test preposition usage, but some will hinge on your knowledge of the appropriate verb form in context. Getting to the Answer: Desire in this context requires the infinitive verb form to blend; the error is in (D). The verb in (A) agrees with its singular subject. (B) and (C) are idiomatically correct.

SAT

Practice Test 2
13

11.

12.

13.

14.

section three

15.

section three

Category: Correct as Written Difficulty: High Issues: comparisons, pronouns, idioms Strategic Advice: If you dont spot an error, dont be afraid to choose (E). Getting to the Answer: This sentence is correct as written. (A) is an appropriate comparative phrase. (B) uses the appropriate pronoun to refer to a person. (C) and (D) are idiomatically correct. Category: SubjectVerb Difficulty: Medium Issues: transitions, idioms, verb tenses Strategic Advice: The test maker will commonly put a plural object immediately before a verb with a singular subject. Dont be fooled. Getting to the Answer: Although the plural noun powers appears immediately before it in the sentence, the subject of the verb are in (A) is actually the singular balance. (B) creates an appropriate relationship between the two clauses. (C) is in an appropriate tense. (D) is idiomatically correct. Category: Modifiers Difficulty: Low Issues: pronouns, verb tenses, idioms Strategic Advice: Adverbs modify verbs, adjectives, or other adverbs, whereas adjectives modify nouns and pronouns. Getting to the Answer: The adjective harsh in (A) incorrectly modifies the verb treated, which requires the adverb form harshly. (B) is the appropriate pronoun to refer to Coach Parks, since he is a person. (C) is an appropriate verb tense in context. (D) is idiomatically correct. Category: Verb Tenses Difficulty: Low Issues: idiom Strategic Advice: Make sure underlined verb tenses properly express the time frames referred to. Getting to the Answer: Since the first part of the sentence refers to something that happened in the past, the present verb cause in (B) should be caused. (A) is idiomatically correct. (C) is proper use of the infinitive. (D) uses an appropriate verb tense in context. Category: Transitions Difficulty: Medium Issues: modifiers, idioms, verb tenses Strategic Advice: Transition words must express the correct relationship between sentences or clauses. Getting to the Answer: As written, the transition word but incorrectly suggests a contrast between the two clauses. Here, the second clause adds information to the first; the conjunction and would be appropriate.

16.

17.

18.

19.

SAT

Practice Test 2
14

(B) appropriately uses an adverb to modify a verb phrase. (C) is idiomatically correct. (D) correctly uses the past tense to refer to a completed action. 20. C Category: Modifiers Difficulty: Medium Issues: subjectverb, idioms Strategic Advice: Adjectives cannot modify other adjectives; youll need an adverb to do that. Getting to the Answer: (C), since its modifying the adjective small, should be the adverb form disappointingly. (A) and (B) agree with their subjects. (D) is idiomatically correct usage. Category: Ambiguity Difficulty: Medium Issues: idioms Strategic Advice: In ISE and IS sentences, each pronoun must have a clear and unambiguous antecedent. Getting to the Answer: Here, there is no antecedent for the pronoun they; (D) contains the error. (A), (B), and (C) are all idiomatically correct. Category: Idioms Difficulty: Medium Issues: subjectverb Strategic Advice: Although most SAT Idiom questions will hinge on preposition usage, some will require you to know the appropriate verb forms. Getting to the Answer: The verb phrase has voted in this context requires the infinitive verb form to be correct; (B) should read to require. (A) agrees with its singular subject. (C) correctly uses the infinitive with the verb require. (D) is idiomatically correct. Category: Parallelism Difficulty: Medium Issues: idioms, modifiers Strategic Advice: Always make sure that related nouns are in agreement. Getting to the Answer: The singular noun advocate in (C) does not agree with the compound noun Nanu and Joanna; since there are two of them, they would have to be advocates. (A) uses an appropriate idiom in context. The adverb in (B) correctly modifies a verb. (D) is idiomatically correct. Category: SubjectVerb Difficulty: Medium Issues: modifiers, comparisons Strategic Advice: A verb must agree in number with its subject noun, which will often not be the noun closest to it in the sentence. Getting to the Answer: The subject of the verb show is not the plural towns

SAT

Practice Test 2
15

21.

22.

23.

24.

section three

section three

(which is the object of the preposition across) but the singular observation; the error is in (A). The verb in (B) agrees with its plural subject. (C) appropriately uses an adverb to modify the adjective similar. (D) creates a correctly constructed comparison between the roads in Kansas and those in New Jersey. 25. E Category: Correct as Written Difficulty: Medium Issues: comparisons, modifiers, subjectverb Strategic Advice: Expect to encounter between five and eight Writing questions without errors on your SAT. Getting to the Answer: This sentence is correct as written. (A) properly uses the comparative phrase such as. (B) correctly uses an adverb to modify a verb. (C) agrees with its plural subject, sports. (D) properly uses the comparative form to compare two items: sports and synchronized swimming. Category: Comparisons Difficulty: Medium Issues: subjectverb, parallelism, idioms Strategic Advice: Make sure comparisons are logically, as well as grammatically, correct. Getting to the Answer: As written, (D) incorrectly suggests that swimming is not a sport. To make the comparison logical, (D) should read any other sport. The verb in (A) agrees with its plural subject. The noun in (B) is correctly plural, since citizens would presumably move to more than one area. (C) is idiomatically correct usage. Category: SubjectVerb Difficulty: High Issues: idioms, modifiers Strategic Advice: And is the only conjunction that creates a compound subject requiring a plural verb. Getting to the Answer: Here, the two nouns notes and beat are joined by neithernor, so the plural verb are in (D) should be is. (A) and (B) are idiomatically correct. (C) appropriately uses an adjective to modify a noun. Category: Comparisons Difficulty: Medium Issues: idioms Strategic Advice: Make sure comparisons are logical. Getting to the Answer: As written, other writers in (C) are illogically compared to the saga ofRabbit Angstrom. (A), (B), and (D) are all idiomatically correct.

26.

27.

28.

SAT

Practice Test 2
16

29.

Category: Idioms Difficulty: High Issues: subjectverb Strategic Advice: Learn common idioms before Test Day. Getting to the Answer: The infinitive to be in (D) is idiomatically incorrect; in this context, as or as being would be correct. (A) agrees with its singular subject. (B) and (C) are idiomatically correct. Category: Run-ons and Fragments Difficulty: Medium Issues: transitions Strategic Advice: More than one answer choice may create a grammatically correct sentence. Your job is to select the one that makes the most sense in the context of the paragraph. Getting to the Answer: The first sentence introduces the idea that Madison is known for one thing. Sentence 2, which is a fragment, identifies the one thing for which Madison is known. A colon is the best way to connect these two ideas. (A), (B), and (E) fail to properly relate the ideas in the sentence. (D) creates a run-on sentence. Category: Passives Difficulty: Medium Issues: wordiness, verb tenses Strategic Advice: Context is important in Improving Paragraphs questions. If more than one choice seems to be correct, take a look at the surrounding sentences. Getting to the Answer: On the SAT, the passive voice will be incorrect if the same idea can be clearly expressed with an active verb. (C) corrects the passive without introducing any additional errors. (A) and (B) do not address the error. The verb tense in (D) fails to properly sequence the events discussed. (E) introduces a verb tense that is inconsistent with the rest of the passage. Category: Transitions Difficulty: Low Issues: none Strategic Advice: When the issue is Transitions, check for relationships. Does this sentence emphasize what came before? Does it provide a contrast or an example? Getting to the Answer: Sentence 7 contrasts with sentence 6; the writers attitude toward Madisons tendency to be late is different from the attitude of most of Madisons friends. Only (B) uses a contrasting transition. (A) is a transition used to emphasize something. (C) is a transition used to introduce an explanation or illustration. (D) and (E) both indicate similarity.

SAT

Practice Test 2
17

30.

31.

32.

section three

33.

section three

Category: Reading Comprehension Difficulty: Low Issues: none Strategic Advice: Questions like this one call on your Reading Comp skills. When youre asked to omit a sentence, keep the main topic of the paragraph and passage in mind. Getting to the Answer: This paragraph is all about how the writer has tried to help Madison be punctual. The amount of time Madison takes to study is not relevant to the topic. The other sentences either state the main idea of the paragraph or are directly related to it. Category: Correct as Written Difficulty: Medium Issues: run-ons and fragments, transitions, verb tenses Strategic Advice: Dont work too hard to find an error. If the sentence seems to be grammatically correct and it makes sense in the context of the paragraph, then (As it is now) is probably the right choice. Getting to the Answer: Sentence 14 emphasizes the idea introduced in sentence 13, that Madison is not thoughtless or inconsiderate. That makes Indeed the best transition. (B) creates a sentence fragment. (C) and (D) indicate a contrast where there isnt one. The past tense verb in (E) is inconsistent with the rest of the passage. Category: Reading Comprehension Difficulty: High Issues: none Strategic Advice: About half of your Improving Paragraph questions will call on the skills you learn in Reading Comp. Getting to the Answer: In this paragraph, the writer weighs Madisons negative habit of being late against her positive actions of remembering birthdays, sending upbeat cards, and cheering up friends. (B) is the most accurate description of the writers strategy. The strategies indicated in the (A), (C), (D), and (E) are not used in the paragraph.

34.

35.

SAT

Practice Test 2
18

Section 4 (Critical Reading)


1. E Category: Cause-and-Effect Difficulty: Low Strategic Advice: Pay attention to Keywords such as cause. Getting to the Answer: The blank will relate to gray steel cages and white walls, but the Keyword lack tells us that well be missing something. Predict something positive that would work with mental, such as stimulation, for the blank. (A) A lack of mental latency is awkward and doesnt relate to the resulting boredom. (B) Mental extension sounds awkward in context. (C) A lack of complacence would result in a favorable situation, which is not what youre looking for. (D) Plain surroundings would not lead to a lack of mental confusion. latency: a period of inactivity complacent: unconcerned; self-satisfied Category: Definition Difficulty: Medium Strategic Advice: Pay attention to sentence structure. Getting to the Answer: Someone who doesnt have a strong opinion probably doesnt care much about the election, so predict a word like uninterested. (B) Tangible is one of those absolute words that cant be qualified: something is either tangible or its not; it wouldnt be quite tangible. (C) Someone without a strong opinion would not be described as abandoned. (D) While autonomous relates to government, it doesnt make sense in context. (E) If you were redundant about voting, it would mean you voted twice, which makes no sense in the sentence. passive: inactive; unassertive tangible: able to be touched autonomous: independent redundant: superfluous Category: Contrast Difficulty: Low Strategic Advice: Begin your work with whichever blank seems easier to you, paying special attention to Keywords like while, which indicates contrast. Getting to the Answer: Start with the first blank: calculators are fast and -------. This blank will have something to do with fast, so predict something like quick or easy. The second blank contrasts with the first blank, so predict something like hard or tough. (A) A calculator cannot be generous, so eliminate this choice. (C) If a calculator were prohibitive, it would not be a fast way of doing math. (D) A calculator cannot be solitary, and convenient does not contrast with fast. (E) Easy works for the first blank, but enjoyable does not contrast with it. efficient: done without waste daunting: discouraging prohibitive: tending to discourage solitary: alone

SAT

Practice Test 2
19

2.

3.

section four

4.

Category: Cause-and-Effect Difficulty: Low Strategic Advice: A Keyword like because will indicate a cause-and-effect relationship within the sentence. Getting to the Answer: Starting with the second blank, a bear would need to create or save fat to keep warm. Predict something like willing or able for the first blank. (A) Squander means to waste, so this choice is the opposite of your prediction. (B) Retain matches your prediction, but if the bear was reluctant, it would be cautious about saving fat. (C) To absorb fat is not the same thing as saving it, and free doesnt make sense in context. (E) A bear that is unwilling to conserve fat is exactly the opposite of what youre looking for. squander: waste reluctant: hesitant conserve: to avoid waste Category: Cause-and-Effect Difficulty: High Strategic Advice: On tougher Sentence Completions, begin your work with whichever blank seems easier to you, paying special attention to Keywords such as once and context clues that reveal underlying structure. Getting to the Answer: The athletes strength could either return or diminish, but if his strength returned, he would want to begin the race again. If his strength diminished, he would not want to race. The Keywords reclaim the title indicate that the pair will have a positive charge. (A) Debunk has a negative charge and buttress has a positive charge, which doesnt match your prediction of matching positive charges. (C) Revoked and relinquish are both negative. (D) Impugned and repudiate are likewise both negative. (E) Vitiated and avoid are also both negative. debunked: disproved buttress: support recommence: begin again revoked: took away relinquish: give up impugned: attacked repudiate: reject the validity of vitiated: corrupted

section four
5.

Robert Tools
6. D Category: Inference Difficulty: Medium Strategic Advice: Remember to read the line before and after the cited text to understand the whole picture. Getting to the Answer: If Tools decided to have the surgery rather than to stay home and die, you can predict that he believed in the possibility of a positive outcome.

SAT

Practice Test 2
20

SAT (A) Out of Scope; Tools did not manipulate doctors into performing the surgery or Practice Test 2 behave in a manner that could be described as conniving. 21 (B) Out of Scope; graceful and poetic are poor descriptors for Tools straightforward language. (C) Opposite; it does not make sense that Tools would feel angry or defeated after surviving an experimental surgery. (E) Distortion; while you might read enthusiasm into Tools lines, nothing in the cited text supports enchantment.

7.

Category: Global Difficulty: Low Strategic Advice: The word primarily signals a Global question, so you know a good prediction will depend on understanding the main idea of the passage. Getting to the Answer: The passage describes some of Tools medical history and the progression of his treatment for heart failure. It also mentions the celebrity gained by physicians and patient alike. (A) Misused Detail; the passage tells you that a traditional heart transplant wasnt an option for Tools but never compares or contrasts the similarities and differences between an artificial and traditional heart transplant. (B) Opposite; the passage states Tools never intended to make history, implying that he did not make a conscious decision to pursue fame. (C) Out of Scope; in the passage, Tools is described as unfit for a traditional heart transplant, but nowhere in the passage is there a discussion of the ideal transplant patient. (E) Out of Scope; Tools family and death are never discussed in the passage, and his occupation is mentioned to imply that his life was rather ordinary before his surgery.

The Heros Journey


8. B Category: Function Difficulty: High Strategic Advice: On Function questions, ask yourself how the cited phrase contributes to the authors overall purpose. Getting to the Answer: In the sentence following the phrase, the author states that, on closer inspection, this story is not the novelty or innovative story that the huge following of loyal fans seems to think it is. Thus, cutting-edge helps to highlight a discrepancy between two lines of thinking. (A) Distortion; the authors main idea is actually the direct opposite of this answer choicehe believes that the movies plot is actually age-old. (C) Misused Detail; the author discusses similarities later on in the paragraph but not here. (D) Misused Detail; the author tells us that the heros journey is an age-old, not a cutting-edge, motif. (E) Out of Scope; the author never indicates that he finds these movies less appealing than other fans do. Category: Detail Difficulty: Low Strategic Advice: For Detail questions such as this one, youll be able to pull your

9.

section four

section four

prediction straight out of the given lines. Getting to the Answer: In the final sentence, the author uses King Arthur and Odysseus, along with Luke Skywalker, to highlight the similarities between stories based upon the heros journey. Your prediction should reflect this. (A) Distortion; there is nothing particularly symbolic about the language the author uses in the final sentencehe basically sticks to straightforward statement. (B) Opposite; the author is actually discussing the research of other scholars, as opposed to his personal opinion. (D) Distortion; the author makes straightforward observations in the final sentence, none of which are ironic. (E) Distortion; the authors tone is generally academic and serious.

Natashas Letters
10. C Category: Vocab-in-Context Difficulty: Medium Strategic Advice: Vocab-in-Context questions can be treated similarly to Sentence Completions. Think of a word or phrase that would mean the same thing within the sentence and compare it to the answer choices, looking for the best match. Getting to the Answer: You know that this light sweetness is what is being conjured, but more importantly, you need to see where it is being applied. The sentence starts by talking about the authors visceral reactions, so this light sweetness describes the particular type of reaction. Therefore, the correct answer choice would have to mean something like produces. (A) Conjure can mean to make something appear, but not in this case. (B) The light sweetness is in reference to the reaction, not the handwriting itself. (D) The letter produces this response; it does not eliminate it. (E) This would mean that the letter is asking the narrator to react, rather than eliciting a gut feeling (which the adjective visceral describes). Category: Inference Difficulty: High Strategic Advice: This Inference question requires understanding the reasons behind the narrators conclusion. What clues was the narrator given? How did the narrator classify them? Getting to the Answer: The only specific clue the narrator gives is that Natashas father was speaking in a different manner than usual, one that the narrator believed meant something was wrong. Look for an answer choice that reflects the authors use of intuition, as the narrators conclusion was not based on any explicit information. (B) Out of Scope; you dont know this from anywhere in the passage. (C) Opposite; you can infer from the passage that the narrator did not understand the words, focusing instead on the subtleties of tone. (D) Out of Scope; you do not have any evidence of this from the passage. (E) Misused Detail; the incident in question occurred before Natasha and the narrator began exchanging letters. Category: Inference Difficulty: Medium Strategic Advice: In this Inference question, its important to understand the logic

11.

Practice Test 2 12.


22

SAT

SAT of the passage selection before matching it to an analogous answer choice. Practice Test 2 Getting to the Answer: The narrator talks about his approach to math problems, 23 one in which he works to understand the nature of the problem rather than merely looking for the answer. He wants to know the internal workings rather than simply to obtain the result. Look for an answer that follows this logic. (A) Out of Scope; the selection deals with comprehension, not with the other persons openness. (B) Distortion; while the narrator had hoped to be Natashas confidante, he concluded something different, and this selection has nothing to do with the narrator being trustworthy. (D) Misused Detail; this relates to an earlier portion of the passage. (E) Distortion; while the narrator does, in fact, seem to be very observant, this doesnt relate to the analogy to which the question stem refers.

13.

Category: Detail Difficulty: High Strategic Advice: This Detail question requires both identifying the narrators original goal and matching it with an answer choice that may phrase it differently. Getting to the Answer: The narrator initially wants to be Natashas confidante and to know that she would not disclose the information given in her letters to anyone else; predict that the narrator wants Natasha to hold him in higher esteem than she does others. (A) Distortion; he wanted to become a confidante rather than to find out that specific fact, and he never pressed Natasha for details on the subject. (B) Distortion; while the narrator eventually did learn to appreciate these subtleties, he did not set out to do this at first. (C) Out of Scope; it is never explicitly mentioned that Natasha is secretive or that the narrator wishes to understand her reasons. (E) Out of Scope; this is never mentioned in the passage. Category: Inference Difficulty: Medium Strategic Advice: Be sure to keep straight who said what on Inference questions such as this one, which asks you to draw conclusions about the narrator in particular. Getting to the Answer: The narrator specifically mentions the handwriting he would hope to see and states that the handwriting would be of more interest to him than the facts contained in the letter. Look for an answer choice that reflects this. (A) Out of Scope; it is never mentioned why the two no longer write to each other or whether or not the narrator believes this to be Natashas fault. (B) Distortion; the handwriting he describes at the end is not the same as the elegant script he described earlier. (C) Opposite; the narrator is less concerned with facts and more concerned with his intuition telling him that Natasha is doing well. (D) Out of Scope; the narrator does not bring up the type of content he hopes to see in her letter.

14.

section four

James Joyce Paired Passages


15. B Category: Inference Difficulty: High Strategic Advice: You should have already noted the main ideas of the passages, so all you need to do here is think about what statements could hinder the main idea of Passage 1. Getting to the Answer: The main idea of Passage 1 is that Joyce, despite, or even because of, the specific local detail in his work, has managed to create works of universal truth and appeal. Predict that the correct choice will somehow suggest a lack of ability to generate such widespread appeal. (A) Misused Detail; though this author briefly mentions historical events in novels, the idea is not at all central to his point. (C) Opposite; this might actually help the authors point, as it speaks to universal human ideals. (D) Opposite; as above, this statement might actually help the authors point. (E) Out of Scope; this point is rather tangential to the aims of the passage, neither helping nor hurting the authors arguments. Category: Vocab-in-Context Difficulty: Medium Strategic Advice: Dont be fooled by familiar words; the challenge of questions like this is in the particular context, not the vocabulary itself. Getting to the Answer: Here, the author uses grand to describe important historical events, so you should predict something like significant. (B) Distortion; although the scope of the historical events is large, the author is not actually making a claim about physical size. (C) Misused Detail; this is an alternate definition of grand that does not have much application to historical events. (D) Misused Detail; again, this definition of grand is not really appropriate in this context. (E) Out of Scope; this doesnt make sense in context. Category: Detail Difficulty: Medium Strategic Advice: When a question is phrased in this way, paraphrase the authors main point in the quoted lines to get your prediction. Remember that the answer choices may be phrased in more general language. Getting to the Answer: Here, the author claims that Joyces level of detail actually helps his work achieve greater appeal. Use such a paraphrase as the basis for your prediction. (A) Misused Detail; though Dublins geography is discussed earlier, it is not the main focus of discussion at this point in the passage. (B) Opposite; the author of Passage 1 mostly describes Joyces subject matter as mundane. (D) Distortion; though Dublin could perhaps be an exotic locale to some, this is not the point the author is making here. (E) Misused Detail; this is a viewpoint more in line with Passage 2 than Passage 1.

section four
16.

17.

SAT

Practice Test 2
24

18.

Category: Inference Difficulty: High Strategic Advice: Use a paraphrase as your prediction, but keep in mind that the answer choices may be written in more general language. Getting to the Answer: Here, the quoted phrase talks about escaping the boundaries of time and place to concentrate on greater truths that are independent of these factors. Expect to find a similar paraphrase among the answer choices. (A) Opposite; the author is talking about revealing universal truths, not specific details. (C) Distortion; although ignoring setting gets close to the right idea, focusing on character is not under discussion at this point. (D) Out of Scope; this author never claims that Joyce wrote in such a way as to obscure information. (E) Misused Detail; although this is an idea in keeping with this passage, it is not the particular meaning of the quoted phrase from the question stem. Category: Global Difficulty: Medium Strategic Advice: Since this question does not reference a specific point in the passage, expect the answer to deal with one of the passages main overall points. Getting to the Answer: One of the main criticisms in Passage 2 is that Joyces subject matter is more base and common than that of earlier authors. Use this idea as the foundation for your prediction. (B) Out of Scope; the author of Passage 2 never questions Joyces factual knowledge. (C) Opposite; if anything, Passage 2 argues that Joyce is too unlike his predecessors. (D) Out of Scope; the author of Passage 2 never makes claims about the elegance of Joyces prose. (E) Out of Scope; this passage never mentions the distinction between Joyces short stories and his novels. Category: Global Difficulty: High Strategic Advice: If you dont know all of the vocabulary, remember that you can nevertheless rule out incorrect choices and make an informed guess. Getting to the Answer: The correct choice is likely to involve the authors main claim about Joyces undignified writing. Predict that the correct answer choice will mean an undignified person. (A) Out of Scope; the author never mentions vanity as a criticism of Joyce. (B) Out of Scope; though others might call Joyce an activist for Ireland, the author of Passage 2 makes no such claim. (C) Out of Scope; the author of Passage 2 is rarely complimentary to Joyce, making this an unlikely choice. (D) Out of Scope; a mystic is someone concerned with religion or the occult, a choice inappropriate for this passages discussion. Category: Inference Difficulty: Medium Strategic Advice: This is a fairly straightforward question; to make your prediction, just summarize the authors main point in the given sentence.

SAT

Practice Test 2
25

19.

20.

21.

section four

section four

Getting to the Answer: In the final sentence, the author makes the claim that Joyce possessed the skill to have become a great writer, had he lived in an era more conducive to dignified writing. Use this as your prediction. (A) Out of Scope; such a claim is never made in the passage and particularly not in the final sentence. (C) Opposite; actually, the author suggests that Joyces prose style was significantly strong and that all he needed was more refined subject matter. (D) Out of Scope; this passage never mentions Joyces need to revise his novels. (E) Out of Scope; this author never claims that Joyce was out of touch with his culture. 22. E Category: Inference Difficulty: High Strategic Advice: Keep in mind the authors general attitudes towards Joyce as you assess their tones at these particular points in the passages. Getting to the Answer: In Passage 1, the author is complimenting the great degree of detail Joyce uses in describing Dublin, whereas the author of Passage 2 is expressing disgust at having to read the details of what he considers vulgar or insignificant acts. Use these tones as the basis for your predictions. (A) Distortion; although both choices here get the general charge right, neither word is quite appropriate to the specific tone of each author. (B) Out of Scope; neither author can really be said to express these attitudes in the passages. (C) Distortion; again, this choice gets the general positive/negative aspects of tone right, but the specifics arent a good match with each authors attitude. (D) Opposite; glee is contrary to the second authors tone. Category: Global Difficulty: Medium Strategic Advice: Although the question stem asks you to compare the passages, all you really have to do is summarize each authors opinion separately. Getting to the Answer: Passage 1 is generally favorable towards Joyce, commending him for evoking many universal truths, while Passage 2 is mostly unfavorable, viewing him as undignified. Use this as the basis of your prediction. (A) Opposite; the author of Passage 1 actually considers Joyce a great author, and inaccuracy is not really a criticism that Passage 2 employs. (B) Opposite; similarly, this is incorrect because the author of Passage 1 promotes Joyces significance as a writer. (C) Distortion; while Author 2 does mention Joyces skill, Author 1 never laments any aspect of the writing nor deems it irrelevant. (E) Distortion; although the general charge is in the right direction for both, neither author makes these particular claims about Joyces writing.

23.

SAT

Practice Test 2
26

Section 5 (Math)
1. B Category: Ratios Difficulty: Low Strategic Advice: Keep your dollars and cents straight. Getting to the Answer: First determine how many cans Sienna can buy for $15.00, then subtract the number of cans she already has from the number of cans she can buy. ($15.00) 1 can = 25 cans $0.60

SAT

Practice Test 2
27

25 cans 20 cans already in her basket = 5 more cans 2. A Category: Equations Difficulty: Medium Strategic Advice: Remember to completely answer the question asked. s Getting to the Answer: First, find the value of p = . Then plug in the values of s, 3 r, and p and solve for n. s p= 3 9 p= 3 p=3 s2r = 10p n3 (9)2(10) = 10(3) n3 (81)(10) = 30n3 810 = n3 30 27 = n3
3

27 =

n3

3=n 3. B Category: Exponents Difficulty: Low Strategic Advice: There are many ways to simplify radicals. Think flexibly, and keep the Kaplan strategies in mind. Getting to the Answer: To solve algebraically, simplify the left side of the equation using your calculator.
3

729 = x 2 9 = x2 3=x

To Backsolve, start in the middle. Note that its easier to calculate (x2)3 and see if it

section five

section five

equals 729 than it is to calculate 729. (C) x = 4 42 = 16 163 = 4,096 Thats too big, so try a smaller number. (B) x = 3 32 = 9 93 = 729 Thats exactly what youre looking for, so you can stop. (B) is correct. 4. B Category: Probability Difficulty: Low Strategic Advice: Probability is the number of desired outcomes over the total number of possible outcomes. Getting to the Answer: There are four slices with onions and peppers, four slices with onions and sausage, four slices with peppers and sausage, and four slices with onions, peppers, and sausage. Thus, there are 4 + 4 + 4 = 12 slices with onions out of 16 total slices. The probability of selecting a slice with onions is 5. C 12 3 = . 16 4

Category: Data Analysis Difficulty: Medium Strategic Advice: Study the chart to find the information you need to solve the problem. Getting to the Answer: The top of the column for 1991 is about halfway between 6,000 and 8,000, so enrollment in 1991 was about 7,000 students. Now, check the answer choices: (A) Enrollment in 1992 was about 15,000 and enrollment in 1993 was about 12,000. Average enrollment for 19921993 was 15,000 + 12,000 27,000 = = 13,500 7,000. 2 2

(B) Enrollment in 1994 was about 10,000. Average enrollment for 19931994 was 12,000 + 10,000 22,000 = = 11,000 7,000. 2 2 (C) Enrollment in 1995 was about 4,000. Average enrollment for 19941995 was 10,000 + 4,000 14,000 = = 7,000, which is the same as the enrollment in 1991. 2 2 You could also solve this problem graphically.
Number of Students 16,000 14,000 12,000 10,000 8,000 6,000 4,000 2,000 0 1991 1992 1993 1994 1995 1996 1997 Average

SAT

Practice Test 2
28

The tops of the columns representing the averages are midway between the tops of the neighboring columns. The column for average enrollment for 19941995 is the same height as the column for enrollment in 1991, so (C) is correct.

6.

SAT Category: Number Lines Practice Test 2 Difficulty: Medium 29 Strategic Advice: Remember the rules for subtracting and adding negative numbers. Getting to the Answer: Any variable to the right of the 0 is positive; any variable to the left is negative. A positive number minus a negative number always equals a positive number. Subtracting a negative number is the same as adding a positive number with the same absolute value. Picking Numbers can make this easier to see. Say a = 5, b = 3, c = 2, d = 1, e = 4, and f = 7. (A) e f = 4 7 = 3 (B) b 0 = 3 0 = 3 (C) a b = 5 (3) = 2 (D) c d = 2 1 = 3 (E) f a = 7 (5) = 12 The only positive number is (E).

7.

Category: Polygons Difficulty: Medium Strategic Advice: Break trapezoids into triangles and rectangles so that you can use familiar formulas to find missing lengths or areas. Getting to the Answer: The perimeter of a figure is the distance around the outside. You know all the side lengths of this trapezoid except for AB. To find AB, add a line that passes through B and is parallel to CD.
A 10 H 24 15 D 24 15 C 26 B

8.

Category: Averages Difficulty: Medium Strategic Advice: In some Averages questions, you will only need the sum of the terms. Getting to the Answer: In order for the average of both teams jersey numbers to remain the same, the sum of each set of traded players jersey numbers must be

section five

Since BH is parallel to CD and perpendicular to AD, BCDH is a rectangle. Therefore, BH = CD = 24 and DH = BC = 15. Then, AH = AD DH = 25 15 = 10. You can see that ABH is a right triangle with legs of length 10 and 24. Thats a multiple of a 5 : 12 : 13 Pythagorean Triplet, so the hypotenuse must be 26. Even if you didnt see that, you could use the Pythagorean Theorem to find the missing side. a2 + b2 = c2 2 + 242 = AB2 10 100 + 576 = AB2 676 = AB2 26 = AB Now you know all the sides of trapezoid ABCD, so you can say that its perimeter is 25 + 24 + 15 + 26 = 90.

section five

equal to each other. Team A trades players 10, 14, and 26, so the sum of their jerseys is 10 + 14 + 26 = 50. Therefore, the sum of the jersey numbers of the Team B players that are traded must also equal 50. (A) 4 + 12 + 16 = 32 (B) 28 + 32 + 36 = 96 (C) 4 + 8 + 12 = 24 (D) 18 + 20 + 24 = 62 (E) 6 + 16 + 28 = 50 The only set of jerseys that sums to 50 is (E), so (E) is correct. 9. A Category: Data Analysis Difficulty: Medium Strategic Advice: Eliminate the graphs that are not consistent with the given information. Getting to the Answer: Since the employees monthly salary goes up after the first three years, you can eliminate (B) right away. The salary is raised once a year; it does not change with time continuously after the first three years, so (D) and (E) are wrong. The salary increases by 5% of the original amount every year after the third. In each graph, the original salary is $2,000, so the increase each year is 0.05($2,000) = $100. Thats the amount shown in (A). You could also use the fact that the original salary is 80 percent of the salary after seven years to distinguish between (A) and (C). The ending salary in (A) is $2,500 dollars. 80% of that is 0.8($2,500) = $2,000, exactly where the graph starts. (A) is correct. Category: Triangles Difficulty: Medium Strategic Advice: Remember that any triangles interior angles sum to 180. Getting to the Answer: Since ABC is a right triangle with a hypotenuse twice the length of the shorter leg, it is a 306090 triangle. (The sides of a 306090 triangle are in the ratio x : x 3 : 2x.) ABC measures 60 andBCA measures 30, since they are opposite the longer leg and the shorter leg, respectively. ABC also measures (k + k), or 2k. Therefore, k equals 30. Use the fact that the interior angles of BCD add up to 180 to find j. BCA + j + k = 180 30 + 30 + j = 180 j = 120 Category: Functions Difficulty: Medium Strategic Advice: Work from the inside out with multiple functions. Getting to the Answer: For each answer choice, plug in the numerical value inside the parentheses for x in g(x). Then use the value determined for g(x) for x in f(x). (A) g(6) = 6 3 = 9 f(9) = (9)2 + 4 = 81 + 4 = 85 Since this is greater than 75, you can stop here. (B) g(5) = 5 3 = 8 f(8) = (8)2 + 4 = 64 + 4 = 68 (C) g(4) = 4 3 = 7 f(7) = (7)2 + 4 = 49 + 4 = 53 (D) g(4) = 4 3 = 1 f(1) = (1)2 + 4 = 1 + 4 = 5 (E) g(5) = 5 3 = 2 f(2) = (2)2 + 4 = 4 + 4 = 8

10.

11.

SAT

Practice Test 2
30

SAT The only answer choice greater than 75 is (A). You could save some work by Practice Test 2 noticing that f(g(x)) will be largest when g(x) has the largest absolute value, whether 31 its positive or negative. Since g(x) = x 3, it has the largest absolute value when x is a negative number with a large absolute value. (A) has the value of x with the largest absolute value of any of the answer choices.

12.

Category: Angles Difficulty: Medium Strategic Advice: When you see a line cross two parallel lines, remember that corresponding angles are equal. All the acute angles equal each other and all the obtuse angles equal each other. Getting to the Answer: Since d and b are corresponding angles, they are equal. Use the fact that supplementary angles (angles that make up a straight line) add up to 180 to find b. f + d = 180 f + b = 180 5b + b = 180 6b = 180 b = 30 All the acute angles are equal, so b = d = e = 30. All the obtuse angles are equal and supplementary to the acute angles, so a = c = f = 150. Evaluate each answer choice to see which one is also equal to 150. (A) c + f = 150 + 150 = 300 (B) e b + c = 30 30 + 150 = 150 (C) d f = 30 150 = 120 (D) e 2f = 30 2(150) = 270 b 30 (E) = =5 6 6 Only (B) is equal to a. Category: Solids Difficulty: Medium Strategic Advice: Remember, the equations for the volumes of cylinders and rectangular prisms are in the reference information at the front of the section. Getting to the Answer: Find the volume of milk in the pitcher. Then, determine how high that volume of milk will fill the rectangular carton. Volume of the pitcher: V = r 2h V = (22)(12) V = (4)(12) V = 48 Height of milk in the carton: V = lwh 48 = (4)(4)h 48 = 16h 3 = h

13.

section five

14.

Category: Functions Difficulty: High Strategic Advice: Always make sure you follow PEMDAS. First calculate the value of the expression inside the parentheses, then move on to the rest of the expression. When youre plugging in multiple variables, make sure you put each one in the right place. Getting to the Answer: Start with the expression inside the parentheses, making sure to plug the first number in for t and the second for u: 9 (3 9) = 3(9) + + 32 = 27 + 3 + 9 = 39 3 39 3 (3 9) = 3 39 = 3(39) + + 32 = 117 + 13 + 9 = 139 3 Category: Complex Figures Difficulty: Medium Strategic Advice: Be sure you know the common geometry formulas. They appear in the reference information at the beginning of each test section, but youll be able to work much faster if you dont need to look them up. Getting to the Answer: The area of a square is the length of a side squared. Since the area of square ABCD is 4, the length of each of its sides is 4 = 2. Therefore, AD = 2. Since AD is half of DE, DE is twice AD, or 2(2) = 4. The area of a rectangle is length times width, so the area of DEFG is DE(DG). DE(DG) = 8 4DG = 8 DG = 2 Category: Functions Difficulty: High Strategic Advice: Always pay attention to units. Getting to the Answer: Here, the units change from hours to days. The mouse receives one pellet every 2.5 hours, or mouse will be fed 24d pellets. 2.5 24 pellets per day. During d days, the 2.5

section five
15.

16.

17.

SAT

Practice Test 2
32

Category: Exponents Difficulty: High Strategic Advice: Dont assume that an unknown number is a positive integer. Unless the question tells you otherwise, you need to consider fractions and negative numbers as well. Getting to the Answer: You are looking for the answer choice that will never be true. If you find that an answer choice is true for just one number, you can eliminate it. Pick Numbers for x. Youll need both a positive and a negative integer to eliminate all the wrong answer choices. Start with x = 2. (A) 22 < 23 or 4 < 8. This is true, so you can eliminate it. (B) 22 < 2 or 4 < 2. This is not true, so keep it for now. (C) 23 < 2 or 8 < 2. This is not true, so keep it for now. (D) 23 < 22 or 8 < 4. This is not true, so keep it for now. (E) 22 < 24 or 4 < 16. This is true, so you can eliminate it. Now try a negative number, like x = 2, for the answer choices that have not been eliminated. (B) (2)2 < 2 or 4 < 2. This is not true, so keep it.

SAT (C) (2)3 < 2 or 8 < 2. This is true, so you can eliminate it. Practice Test 2 3 < (2)2 or 8 < 4. This is true, so you can eliminate it. (D) (2) 33 The only answer choice left is (B). For any integer x, x2 will not be less than x. If x is 1 or 0, then x2 will equal x. If x is another positive integer, then x2 will be greater than x. If x is a negative integer, then x2 will be positive and therefore greater than x. (B) is never true, so it is the correct answer.

18.

Category: Equations Difficulty: High Strategic Advice: Work methodically, making sure not to mix up any of the variables. Getting to the Answer: First, substitute 4 for a and x for b into the equation and solve for x: a = 3b 2 4 = 3x 2 6 = 3x 2 = x, so (x, y) is (2, y). Substitute (2, y) into the equation: a = 3b 2 2 = 3y 2 4 = 3y 4 =y 3 Note that (E) is a trapits what you get if you reverse a and b.

19.

Category: Geometric Visualization Difficulty: High Strategic Advice: Once you find a particular number of pieces, you can eliminate any lower answer choices. As long as you can eliminate even one choice, its to your advantage to guess. Getting to the Answer: To get the maximum number of parts, line l should cross both of the existing lines.

2 3

5 7 8

This divides the shaded region into 9 parts. 20. B Category: Complex Figures Difficulty: High Strategic Advice: Many geometry questions will require you to add a line. Getting to the Answer: The perimeter of the rectangle is 22 and OP is 8. Opposite sides of a rectangle are equal, and the perimeter is the sum of the lengths of all four

section five

section five

sides, so you can find OR from this. OR + PQ + OP + RQ = 22 2(OR) + 2(8) = 22 2(OR) + 16 = 22 2(OR) = 6 OR = 3 Draw radius OS to create a right triangle with two legs of length 3.

O 3 R 3

P S

Since ORS is a right triangle with legs of the same length, it is a 454590 triangle with sides in the ratio x : x : x 2. Since x = 3, the hypotenuse is 3 2. The hypotenuse of ORS is also the radius of circle O. The area of a circle is r2, so the area of circle O is (3 2)2 = (3)2( 2)2 = (9)(2) = 18.

SAT

Practice Test 2
34

Section 6 (Critical Reading)


1. E Category: Cause-and-Effect Difficulty: Low Strategic Advice: Sentence logic and Keywords can indicate a cause-and-effect relationship. Getting to the Answer: Start with the second blank, predicting some destructive result of excess rain, such as flooding. The first blank is harder to predict. What sort of region would be necessarily prone to flooding? (A) Rain wouldnt cause tropical regions to freeze. (B) Rain wouldnt cause a marsh to parch; this is, in fact, the opposite of what it would do. (C) Rain in the desert would probably be a good thing for the villages, so eliminate this choice. (D) Rain wouldnt cause anything to aerate, and this would certainly not result in devastation. parch: to dry out aerate: to fill with air Category: Cause-and-Effect Difficulty: Medium Strategic Advice: Based on the Keywords in the sentence, the blanks seem like they would have similar meanings. Getting to the Answer: Beginning with the first blank, Jodie could either have caused the bullys ire or prevented it. Moving on to the second blank, Jodie could have either received months of teasing, or prevented the teasing, respectively. (A) Protecting and inhibited are incorrect in context and lack the necessary relationship youre looking for. (B) Supplanting and disguised are unrelated words and dont make sense in context. (C) Limiting might work for the first blank, but not with satisfied for the second. (D) Deriding and compelled lack a strong relationship. supplanting: replacing provoking: inciting Category: Definition Difficulty: Medium Strategic Advice: Always make a prediction before looking at the answer choices, paying attention to Keywords and sentence logic. Getting to the Answer: If the athletes could no longer play together, the issue must have involved strong, opposing feelings. Predict something like highly debated. (B) A transparent issue wouldnt result in discord. (C) Concordant is the opposite of your prediction, suggesting agreement. (D) A repetitive issue wouldnt necessarily cause disagreement among the players. (E) A rhetorical issue likewise wouldnt result in discord. divisive: resulting in disagreement, or strife Category: Contrast Difficulty: Medium Strategic Advice: Begin your work with whichever blank seems easier to you, keeping in mind that although signals contrast.

SAT

Practice Test 2
35

2.

3.

4.

section six

Getting to the answer: Starting with the first blank, the theory could either seem plausible or implausible. The second blank would then have to be the opposite of the first. Predict either false or true, respectively. (A) Dubious and erroneous both have a negative charge and wont provide the necessary contrast. (B) Based on their prefix in, both incongruous and inconsistent have a negative charge, so they wont work in a sentence that requires contrasting words. (D) Logical and pertinent both have positive charges, so they also fail to provide the contrast youre looking for. (E) Probable closely matches your prediction, but coherent isnt a good opposite. dubious: uncertain incongruous: incompatible pertinent: relevant 5. C Category: Definition Difficulty: High Strategic Advice: Begin your work with whichever blank seems easier to you, especially when clues in the sentence arent immediately suggestive of a strong relationship between the missing words. Getting to the Answer: Beginning with the second blank, what sort of picture would be appropriate to success in the face of adversity? Predict something like powerful or moving. The first blank is harder to predict, but you know that you have a series and that the pieces are short, suggesting something like stories. (A) Since illusory sounds like illustration, this choice may have been tempting, but illusory really means illusion or deception, so it doesnt work in context. (B) Thorough isnt emotionally charged enough to match your prediction. (D) While authentic may work for the second blank, treatises would not be drafted about children surviving a war, and they certainly wouldnt be short. (E) Ambiguous doesnt work in context; youre looking for much more assertive language. missives: letters vignettes: short, descriptive stories ambiguous: unclear Category: Definition Difficulty: High Strategic Advice: Pay attention to sentence structures that define the missing word, such as a list or series that relates back to the blank. Getting to the Answer: The Keywords point to fullness or excess, as the sentence enumerates a list of expensive items. Predict something like collection. (A) Summary is neither positive nor negative and does not make sense in context. (B) Fabrication sounds like fabricate, to invent or create, so fabrication must have something to do with being false and does not match your prediction. (C) The prefix con makes consensus seem like a positive word, but it relates to agreement so doesnt work in this context. (E) Replication makes it sound like the treasure was inauthentic, an idea which is not supported by the context. fabrication: falsehood cache: a concealed store of goods or valuables replication: repetition; copy

section six
6.

SAT

Practice Test 2
36

7.

Category: Cause-and-Effect Difficulty: High Strategic Advice: A sentence will often directly define the missing word. Getting to the Answer: The sentence itself is not difficult in this example, but the choices contain tough vocab, so be especially certain that youve made a strong prediction before evaluating the answer choices. The Keywords easily persuaded directly define the blank, so predict something like controllable. (A) A frolicsome president wouldnt necessarily be easily persuaded. (B) Cantankerous has a negative charge but describes a disagreeable individual, not one who is easily manipulated. (C) Gargantuan relates to size and therefore doesnt work in context. (E) Precipitate can describe moving in haste or without deliberation, so you may be tempted. Remember, though, that youre looking for a necessary relationship with easily persuaded. frolicsome: playful cantankerous: disagreeable gargantuan: huge; giant tractable: controllable precipitate: fast; acting in great haste Category: Definition Difficulty: Low Strategic Advice: Always make a prediction, considering sentence structure and Keywords, before looking at the answer choices. Getting to the Answer: The Keywords map out and fingerprints can lead you to a prediction like design or markings. (A) While a computer may be able to create a model of an eye, the Keywords map out dont work with this choice. (C) Lets hope there arent remnants of your eye! This choice doesnt match your prediction and doesnt make sense in context. (D) Populations likewise doesnt work in the context of the sentence. (E) While an eye does have pigments, you know that the computer must map something analogous to the unique design of fingerprints. remnant: something left behind

SAT

Practice Test 2
37

8.

Athletics Paired Passages


9. C Category: Global Difficulty: Medium Strategic Advice: Focus on the common thread throughout Passage 1; what language seems to be recurring? Getting to the Answer: Author 1 spends nearly every sentence praising the fantastic physical ability of todays athletes, so your prediction should reflect this emphasis on physical excellence. (A) Misused Detail; while Author 1 compares modern athletes to those of the past, this is not his main focus. (B) Opposite; while Passage 2 concentrates on changing values in athletics, Passage 1 makes no mention of it. (D) Opposite; again, it is Passage 2 that focuses on the theoretical moral

section six

consequences of heightened athletic achievement. (E) Out of Scope; neither passage discusses the popularity or marketability of athletes. 10. D Category: Global Difficulty: Low Strategic Advice: For questions about tone, pay special attention both to the language and overall purpose of each authorthese will be the keys to making a good prediction. Getting to the Answer: Both passages represent attempts by the authors to convince the reader of a personal opinion: that contemporary sports are either praiseworthy or deplorable. Since both authors strongly assert their points of view, make a prediction that reflects their lack of objectivity. (A) Opposite; each author is making a strong and overt case for a particular point of view and weighs in heavily with an opinion. (B) Distortion; there is nothing particularly scientific about either authors analysis, since there are few purely factual statements. (C) Distortion; while Author 2 could be described as gloomy or negative, Author 1 is very optimistic about the current state of sports. (E) Opposite; again, each author is making a strong case for his viewpoint, so both tones are partial. Category: Inference Difficulty: Medium Strategic Advice: Inference questions ask you to read between the lines, using information from the passage to draw your own conclusions. Getting to the Answer: In the final sentence of Passage 1, the author shows admiration for the degree to which athletes dedicate themselves to physical excellence; dedication would be a good prediction. (A) Misused Detail; Author 2 discusses the greed of athletes. (C) Misused Detail; Author 2 talks about decline in the context of ethics in presentday athletics. (D) Opposite; Author 1 treats athletes determination very positively. (E) Distortion; while Author 1 certainly thinks that athletes have great physical aptitude, the word does not convey the sense of dedication or commitment that youre looking for. Category: Global Difficulty: Medium Strategic Advice: On Global questions that require general comparisons, first consider what each author is trying to accomplish. Getting to the Answer: Author 1 writes about the unparalleled physical abilities of todays athletes in a very positive light, using words such as devote and excellence. Author 2 acknowledges that the athletes may be more talentedthe quality of play ison the upswingbut goes on to describe how this has been accompanied by a crisis of sorts. (A) Distortion; while both passages recall past eras of sports history, neither indicates anything about the future. (B) Opposite; it is Author 2 who focuses on a disturbing trend, while Author 1 is generally positive about the state of athletics. (C) Opposite; again, it is Author 2 who decries the current direction of sports,

section six
11.

12.

SAT

Practice Test 2
38

while Author 1 glorifies its heroes. (D) Distortion; while Passage 1 does focus mostly on the talent of modern athletes, there is no mention of training methods in Passage 2. 13. C Category: Global Difficulty: High Strategic Advice: For questions testing similarity between passages that generally disagree, youll be able to draw your answer directly from a few specific lines of text. Getting to the Answer: Author 1 thinks very highly of the physical ability of todays sportspeople, and Author 2 agrees that the quality of play is better than before, so you can predict a word that conveys a great degree of talent or ability. (A) Distortion; Author 1 never mentions greed or avarice anywhere in the passage. (B) Opposite; both authors convey the sense that todays athletes possess great ability. (D) Distortion; while both authors would likely describe todays athletes as exhibiting zeal, this description does not apply to physical ability. (E) Distortion; Author 1 does not convey anything controversial.

SAT

Practice Test 2
39

Pharmaceutical Costs
14. B Category: Function Difficulty: Low Strategic Advice: Focus on the paragraph as a wholedont be trapped by details within it. Getting to the Answer: The paragraph references labor, ongoing drug research, and progress. Look for an answer that ties together these clues. (A) Out of Scope; the passage does not address research techniques, only research in general. (C) Misused Detail; the benefits of new medications support the paragraphs main ideathat research is productive. (D) Misused Detail; the author mentions the years of labor by the scientific community to support the theme of scientific research, not to comment on hard work. (E) Distortion; the ambitions of the medical community represent its goals, not its level of competition. Category: Function Difficulty: High Strategic Advice: The text immediately surrounding the citation can help you on this question, but make sure not to stray too far from the citation itself. Getting to the Answer: The phrase before the specific description indicates that the public is tempted by this argument, and the cited text describes the content of the argument as poor but appealing. (B) Misused Detail; those making claims are uninformed, but the author sympathizes with the public that is tempted by the argument. (C) Extreme; though the argument is straightforward, the passage does not prove the effectiveness of direct arguments in all cases. (D) Distortion; those seeking lower drug prices do not believe that their argument is false.

15.

section six

(E) Misused Detail; the citation does not compare these two components but states that the same argument includes both. 16. D Category: Inference Difficulty: Medium Strategic Advice: If the citation is complex, the surrounding text will often provide extra clues. Getting to the Answer: In the sentence preceding the citation, the author calls the beliefs of those seeking lower drug prices false, and claims they are committing a wrong to perpetuate this belief. Your prediction should summarize this. (A) Extreme; the author does not claim that they are immoral, only that they are being unfair to the ill. (B) Out of Scope; the author never addresses the effects of attitude on health. (C) Out of Scope; the author never discusses the effect of illness on susceptibility to argument or persuasion. (E) Opposite; the author makes it clear that the actual wrongs are worse than perceived ones. Category: Inference Difficulty: High Strategic Advice: The phrase In the context of the passage is an important clue that the answer will be more global in nature and not based on any single detail. Getting to the Answer: The central argument of the passage is that drug prices need to be high to help coverresearch and not just production costs. The ability of patent-protected prices to guarantee income to cover research costs is, then, their most important aspect. (A) Distortion; only products and information, not prices, can be privately owned. (B) Misused Detail; though the opposition considers these prices unfair, the passage as a whole indicates that they are essential toprogress and cover past research. (D) Out of Scope; the governments role in setting drug prices is never addressed. (E) Misused Detail; though these prices are daunting, their most important function is to fund research. Category: Detail Difficulty: Medium Strategic Advice: In a passage that expresses a strong opinion, pay special attention to the attitude of the author. Getting to the Answer: Because you know the author favors drug companies and that this paragraph reflects the authors own opinion, you can eliminate answers that are negative towards corporations. On referring to the text, its clear that the author is using the term to describe drug companies corporate nature, nothing more. (A) Distortion; big indicates the nature of the corporation, not the demand for its product. (B) Extreme; though corporations want to see financial fruits, the passage does not imply an insatiable appetite for profit. (C) Out of Scope; the role of corporations in society as a whole is not a theme of the passage. (D) Extreme; though businesses arent financed on goodwill alone, they are never referred to as completely or necessarily immoral.

section six
17.

18.

SAT

Practice Test 2
40

19.

SAT Category: Function Practice Test 2 Difficulty: Low 41 Strategic Advice: You should always reference your text for content, but in passages that present strong opinions, be sensitive to the authors tone as well. Getting to the Answer: This citation appears in a paragraph that criticizes those who wish to lower drug prices. Because the author is strongly opinionated, it is likely that this adjective is being used to challenge or even ridicule proponents of low-cost alternatives. (A) Opposite; the author acknowledges that drugs can be expensive. (C) Distortion; the phrase bargain basement speaks only to price, not to quality. (D) Distortion; the paragraph itself discusses shrinking research funds, but the most important function of this particular phrase is to criticize low prices. (E) Misused Detail; the passage does not compare low-priced drugs to free drugs; it criticizes both.

20.

Category: Vocab-in-Context Difficulty: High Strategic Advice: When the cited sentence offers you few clues, check the surrounding text. Getting to the Answer: From the preceding sentence, you find that only the cost of ingredients and manufacture make up the grossest level of drug production. By combining these two sentences you can see that grossest means the most apparent or obvious. (B) Out of Scope; ingredients and manufacture are not vulgar or offensive. (C) Misused Detail; those calculating drug cost consider prices based on the cost of manufacture and ingredients to be fair, but grossest does not have this meaning. (D) Distortion; though manufacturing may be the most simple step, grossest does not convey this meaning in context. (E) Out of Scope; these particular steps in the process are not ugly or bizarre. Category: Function Difficulty: Medium Strategic Advice: Do not stray too far from the text. Remember, the answer is in the passage. Getting to the Answer: This paragraph criticizes those who support lower drug costs, and this question is meant to portray their thought process. Look for an answer that most directly expresses this fact and is supported by the text. (A) Misused Detail; it is not humanity as a whole, but rather those who support lower-priced drugs, that seek to benevolently dispense cures, and the author never directly discusses individual greed. (B) Distortion; the citation does not question that progress can be owned as much as it questions the right of any individual to own it. (D) Out of Scope; the passage discusses only drug cost, not the much broader topic of general social equity. (E) Opposite; this question is presented mockingly by the author; it is meant to demonstrate the beliefs of, and thus supports, those who do want drug knowledge to be widely shared. Category: Inference Difficulty: Low Strategic Advice: Inference questions that direct you to a specific portion of text still require you to think globally. How does this paragraph fit into the whole passage

21.

22.

section six

in terms of tone and content? Getting to the Answer: The references to benevolence, humanity, good-natured people, and faith all mock those who believe in the feasibility of lower drug costs. The paragraph is used to support the passages main idea that such calculations are idealistic and unrealistic. (A) Misused Detail; the author claims only sarcastically that a drug can be made by humanity, for humanity. (C) Misused Detail; the author believes that income is essentialto progress, and states only sarcastically that good-natured people simply stumble upon new drugs. (D) Out of Scope; though they may benevolently dispense cures, straightforward benevolence is not evaluated as a motivation. (E) Misused Detail; the author believes that innovation comes from research, not faith. 23. E Category: Function Difficulty: High Strategic Advice: Think about how a sentences structure can convey meaning, in addition to its content and context. Getting to the Answer: This sentence follows a critique of idealist values. By proceeding to disregard the character of the researchers, the beginning of the citation dismisses the importance of disposition in science. The right answer will focus on the results of research and ignore more emotional variables. (A) Opposite; the author is not interested in the disposition of researchers, only in enabling them to pursue their studies. (B) Out of Scope; the author does not suggest a relationship between the severity of diseases and the research that should go into them. (C) Distortion; the citation indicates that it is unimportant whether those who found the treatment were good-natured or not. (D) Misused Detail; though a group of people discovered the treatment, the author is unconcerned about the specifics of its discovery. Category: Function Difficulty: Low Strategic Advice: Detailed description is a technique authors often use to strengthen an argument. Think about what main idea this description supports. Getting to the Answer: Jasmines illness is introduced in the passage as an example of a disease that was previously managed poorly but is now effectively treated. The cited text helps to make clear the severity of her disease and the inadequacy of her earlier treatment options. (A) Misused Detail; the fact that Jasmine has a bone marrow disease is less important than the fact that her condition was being treated ineffectively. (B) Distortion; this description focuses on Jasmines battle with her condition, not the medical communitys struggle. (C) Distortion; Jasmine was dependent on blood transfusions, but these treatments were meant to be long-term and Jasmines dependence on them was medically necessary. (E) Out of Scope; although it is likely that Jasmine desired a cure, the passage does not address Jasmines feelings about her condition.

section six
24.

SAT

Practice Test 2
42

25.

Category: Global Difficulty: Low Strategic Advice: When asked to evaluate a paragraph, look not only at its content, but also at its role in the passage as a whole. Getting to the Answer: Because this is the concluding paragraph, it will likely reinforce some main themes of the passage. The author states that the combination is more important to a drug than its specific ingredients and, again, emphasizes the role of research. (A) Distortion; the author claims only that research is at risk of being underfunded if drug prices drop. (B) Misused Detail; though the author states the best way to protect the sick isto protect the industry that heals them, the focus remains on the industry, not the ill. (C) Opposite; the author claims that to think this way is to disregard the role of research. (E) Misused Detail; the author doesnt speak for all businesses, only the drug industry.

SAT

Practice Test 2
43

section six

Section 7 (ExperimentalWriting)
1. C Category: Run-ons and Fragments Difficulty: Low Issues: transitions, wordiness Strategic Advice: Although there are a number of ways to correct a run-on sentence, only one answer choice will do so without introducing additional errors. Getting to the Answer: This sentence is a run-on, since both clauses are independent. (C) correctly combines the clauses by making the second one dependent. (B) does not address the error. (D) leaves the meaning of the second clause incomplete. (E) is unnecessarily wordy. Category: Wordiness Difficulty: Medium Issues: passives Strategic Advice: If you dont spot an error in grammar or usage, look for errors in style. Getting to the Answer: As written, this sentence is unnecessarily wordy. (B) is concise without losing any of the sentences meaning. (C) introduces the passive voice unnecessarily. (D) and (E) are still unnecessarily wordy. Category: Correct as Written Difficulty: Medium Issues: run-ons and fragments, modifiers, style problems Strategic Advice: The SAT frequently uses formal or complex sentence structure to disguise sentences that are correct as written. Getting to the Answer: This sentence is correct as written. (B) and (D) create sentence fragments. In (C), the introductory phrase While working as a patent clerk incorrectly modifies Albert Einsteinspapers. (E) leaves the meaning of the second clause incomplete. Category: Passives Difficulty: High Issues: wordiness Strategic Advice: Passive constructions will nearly always be unnecessarily wordy. Getting to the Answer: This sentence uses the passive the hysteria caused by the broadcast of The War of the Worlds was only short-lived; (D) makes the sentence active and more concise. (B) and (C) are still in the passive voice. (E) is unnecessarily wordy. Category: Style Problems Difficulty: Medium Issues: transitions Strategic Advice: A sentence requires only one subject. Getting to the Answer: The subject of this sentence is the compound Sir Edmund Hillary and Tenzing Norgay; the additional pronoun subject they is incorrect. (D) is

section seven
2.

3.

4.

5.

SAT

Practice Test 2
44

the best revision here. (B) and (C) do not address the error. (E) reverses the cause-and-effect relationship between the two clauses; reaching the summit was the cause, not the effect, of the explorers record. 6. E Category: Run-ons and Fragments Difficulty: Medium Issues: modifiers, style problems Strategic Advice: Most SAT sentences will have multiple nouns and verb forms; identifying a sentences subject and predicate verb will help you recognize fragments and run-ons. Getting to the Answer: As written, this sentence is a fragment. It contains a subject, an unknown soldier, and two modifying phrases, but no predicate verb. (E) creates a complete sentence. In (B), it is the tomb, rather than the unknown soldier, that is Memorialized. (C) is grammatically incorrect. In (D), its the soldier, not the tomb, that sits atop a hill. Category: Run-ons and Fragments Difficulty: Medium Issues: style problems, pronouns Strategic Advice: The ing verb form can never be the predicate verb in a sentence. Getting to the Answer: This sentence is a fragment with no predicate verb. (C), (D), and (E) all correct the fragment error, but (C) is grammatically incorrect, and (D) uses a plural pronoun to refer to the singular orchestra. (B) creates a grammatically incorrect sentence. Category: Ambiguity Difficulty: High Issues: style problems, run-ons and fragments Strategic Advice: This is often used as a pronoun in everyday speech, but without an antecedent, it will be incorrect on the SAT. Getting to the Answer: In this sentence, this is used with no clear antecedent. (B) replaces the ambiguous this with an appropriate noun, this fact. (C) creates a run-on sentence. (D) doesnt make it clear who or what is doing the explaining. (E) misuses the semicolon splice, which can only combine independent clauses.

SAT

Practice Test 2
45

8.

10.

Category: Modifiers Difficulty: High Issues: none

section seven

9.

Category: Parallelism Difficulty: Low Issues: none Strategic Advice: Items in a series or list must be parallel in structure. Getting to the Answer: The first two items in this series are past tense verbs (became and changed), so the third should be as well. Only (E) makes the correction. (B), (C), and (D) fail to create parallel structure.

section seven

Strategic Advice: Generally, an introductory modifying phrase will modify the first noun or pronoun that follows it. Getting to the Answer: As written, this sentence tells you that the meals ingredients are encouraging the experimentation of the customers. Only (E) puts the proper noun, restaurant, after the modifying phrase. (B) and (C) do not address the error. In (D), the opening phrase modifies the diner. 11. C Category: Style Problems Difficulty: Medium Issues: run-ons and fragments, modifiers Strategic Advice: A sentence with a noun subject does not need a pronoun subject as well. Getting to the Answer: Since they here refers to automobiles, it is redundant as part of the subject. (C) eliminates the redundant pronoun, making automobiles the subject of the verb were becoming. (B) creates a sentence fragment, since none of the three clauses is independent. (D) does not address the error. (E) incorrectly makes many Americans the subject modified by the opening phrase of the sentence. Category: Ambiguity Difficulty: Medium Issues: transitions, verb tenses, pronouns Strategic Advice: Although pronouns like it and they are frequently used without antecedents in everyday speech, such usage will be incorrect on the SAT. Getting to the Answer: The only singular noun in this sentence is knowledge, which is not a logical antecedent for it in this context; the error is in (D). (A) correctly expresses the contrast between the ideas in the two clauses. (B) is an acceptable verb tense in context. (C) is appropriate use of the reflexive pronoun myself. Category: Correct as Written Difficulty: High Issues: modifiers, transitions, pronouns Strategic Advice: In complex sentences, remember that you only need to evaluate the four underlined portions for possible errors. Getting to the Answer: This sentence does not contain any errors. (A) and (C) properly use adverbs to modify adjectives. (B) uses an adjective to modify a noun. (D) correctly shows that the second clause is the cause of the first. Category: Pronouns Difficulty: High Issues: transitions, verb tenses Strategic Advice: The test maker disguises pronoun case errors in compounds because they sound wrong when read alone. Read the sentence with just the pronoun to hear the error. Getting to the Answer: Since you wouldnt say After asking I for my passport, the pronoun in (B) should be me. (A) is an appropriate transition word in context.

12.

13.

14.

SAT

Practice Test 2
46

(C) uses the proper possessive pronoun for the compound Christine and I. (D) is an appropriate verb tense for an action that has not yet happened. 15. A Category: SubjectVerb Difficulty: Medium Issues: transitions, pronouns, idioms Strategic Advice: A compound subject requires a plural verb. Getting to the Answer: Here, the subject is the compound The manager and his assistant, so the singular verb in (A) is incorrect. (B) is an appropriately used transition. (C) agrees with its compound antecedent. (D) is idiomatically correct usage in context. Category: Parallelism Difficulty: Medium Issues: idioms, subjectverb Strategic Advice: Related nouns in a sentence must agree in number. Getting to the Answer: The subject of this sentence is the compound, and therefore plural, night-vision goggles and the Humvee. These items have not become a singular consumer product, but rather, consumer products; the error is in (D). (A) and (B) are correct idiomatic usage. (C) agrees with its compound subject. Category: Pronouns Difficulty: Medium Issues: subjectverb, verb tenses, modifiers Strategic Advice: When referring to people, use who or whom. Getting to the Answer: Which, in (B), is incorrect, since it refers to a federal judge. (A) agrees with its plural subject players. (C) uses the proper tense for an action completed in the past. (D) correctly uses an adjective to modify a noun and an adverb to modify the adjective. Category: Comparisons Difficulty: Medium Issues: idioms, subjectverb Strategic Advice: Comparative adjectives are formed by either adding an er or the word more; using both is incorrect. Getting to the Answer: Trendier is already a comparative adjective, so more is incorrect; the error is in (C). (A) and (D) are idiomatically correct. (B) agrees with its singular subject. Category: Pronouns Difficulty: Medium Issues: idioms, transitions Strategic Advice: Check underlined pronouns for agreement with their antecedent nouns. Getting to the Answer: The plural pronoun those in this sentence does not agree

SAT

Practice Test 2
47

16.

17.

18.

19.

section seven

with its singular antecedent, recipe; the error is in (C). (A) and (D) are both idiomatically correct. (B) is an appropriate transition to indicate a causal relationship. 20. E Category: Correct as Written Difficulty: Medium Issues: pronouns, modifiers, idioms, subjectverb Strategic Advice: Eliminate wrong answer choices methodically; remember that between five and eight Writing sentences will have no errors. Getting to the Answer: This sentence is correct as written. (A) is an appropriate use of one as an indefinite pronoun. (B) uses an adverb to modify the verb phrase can...see. (C) properly sets up the idiom bothand. The verb in (D) agrees with its compound subject the ocean and the volcano. Category: Modifiers Difficulty: Low Issues: subjectverb, idioms, verb tenses Strategic Advice: Adjectives can only modify nouns and pronouns; all other parts of speech are modified by adverbs. Getting to the Answer: Here, the adjective easy is being used to modify the verb passed; (D) should be easily. The verbs in (A) and (B) agree with their singular subjects and about in (A) is idiomatically correct in context. (C) makes appropriate use of the infinitive verb form. Category: SubjectVerb Difficulty: Medium Issues: modifiers, pronouns Strategic Advice: Be sure to identify the proper subject noun for each underlined verb; it will rarely be the noun closest to it in the sentence. Getting to the Answer: Even though the plural nouns channels and years are closer to it, the subject of the verb phrase have had is actually the singular explosion. (A) contains the error. (B) properly uses an adjective to modify a noun and an adverb to modify the adjective. (C) agrees with its subject, each. (D) agrees with its antecedent, points of view. Category: Correct as Written Difficulty: Medium Issues: pronouns, subjectverb, verb tenses, modifiers Strategic Advice: Use your elimination skills to identify sentences that are correct as written. Getting to the Answer: This sentence contains no error. (A) agrees with its plural subject, and who is the right pronoun to refer to people. (B) uses a verb tense that is appropriate in context. The pronoun in (C) agrees with its plural antecedent, consumers. (D) correctly uses an adjective to modify a noun.

section seven
21.

22.

23.

SAT

Practice Test 2
48

24.

Category: Pronouns Difficulty: Medium Issues: verb tenses, idioms Strategic Advice: Check underlined pronouns for appropriateness of number and case. Getting to the Answer: Since you wouldnt say them students, the pronoun in (C) should be those. The verbs in (A) and (D) are both in an appropriate tense. (B) is idiomatically correct usage in context. Category: Correct as Written Difficulty: High Issues: idioms, comparisons Strategic Advice: Expect between five and eight sentences in your SAT Writing sections to be correct as written. Getting to the Answer: This sentence is correct as written. (A) correctly uses the ing verb form as a modifier. (B) is an idiomatically correct comparative phrase. (C) is idiomatically correct usage. (D) is correctly in the superlative, since the sentence is comparing the blue whale to all other animals. Category: Diction Difficulty: High Issues: modifiers, pronouns Strategic Advice: A grammatical construction may be correct in one context but incorrect in another. Getting to the Answer: If this sentence read Italy is a country that people truly enjoy, the phrase in (B) would be correct. As written, however, the sentence is grammatically incorrect; (B) should read a country in which. (A) correctly uses two adjectives to modify a noun. The adverb in (C) properly modifies the verb enjoy. The pronoun in (D) is correct for its antecedent, Italy. Category: Comparisons Difficulty: Medium Issues: idioms, transitions, verb tenses Strategic Advice: When comparing two items, the comparative adjective form is correct. Getting to the Answer: Since the experience of only two drivers is compared here, (C) should read less experienced driver. (A) is correct idiomatic usage. (B) creates an appropriate contrasting relationship between the clauses. (D) uses a verb tense that is consistent with the rest of the sentence. Category: SubjectVerb Difficulty: Medium Issues: adjective/adverb, idioms Strategic Advice: Dont mistake the object of a preposition for the subject of a verb; in an SAT sentence, the subject will frequently not be the noun closest to the verb.

SAT

Practice Test 2
49

25.

26.

27.

28.

section seven

Getting to the Answer: The subject of this sentence is the singular knowledge; writings is the object of the preposition of. (B) should read is. (A) properly uses an adjective to modify a noun. (C) and (D) are correct idiomatic usage.

section seven

29.

Category: Correct as Written Difficulty: Medium Issues: parallelism, pronouns, verb tenses, idioms Strategic Advice: Expect between five and eight sentences in your Writing sections to be correct as written. Getting to the Answer: This sentence is correct as written. (A) correctly parallels the other verb in the compound. (B) has a clear antecedent, school. The verb in (C) is consistent in tense with the sentences other verb, have. (D) is idiomatically correct usage. Category: Run-ons and Fragments Difficulty: Medium Issues: wordiness, transitions Strategic Advice: A sentence may have multiple nouns and verbs and still be a fragment. Getting to the Answer: This sentence is a fragment, since it has no independent clause. (A), (B), and (D) all correct the fragment error, but (B) is unnecessarily wordy, and (D) creates an illogical contrast between this sentence and the one before it. (C) and (E) do not address the error. Category: Wordiness/Diction Difficulty: High Issues: none Strategic Advice: Since this question stem asks which revision would NOT improve the sentence, any revision that WOULD improve the sentence can be eliminated as an answer choice. Getting to the Answer: Of the five revisions proposed here, the only one that would not improve sentence 4 is (C), which would only make this extremely wordy sentence even wordier. Category: Reading Comprehension Difficulty: Medium Issues: none Strategic Advice: About half of your Improving Paragraphs questions will require you to use your Reading Comp skills. Getting to the Answer: Sentence 5 emphasizes that Morgan was talented and inventive; (B) is the best description of this sentences function. Although the passage is a personal narrative, there really arent any personal opinions introduced, so (A) can be eliminated. Sentence 5 doesnt introduce any new ideas, so neither (C) nor (E) is correct. Since the passage doesnt contain any hypothetical statements, (D) is also incorrect.

30.

31.

32.

SAT

Practice Test 2
50

33.

Category: Wordiness Difficulty: Medium Issues: reading comprehension, idioms Strategic Advice: Beware of answer choices than change the meaning of the original sentence. Getting to the Answer: In a situation in which is just a longer way of saying when; the best choice here is (B). (A) and (E) eliminate some of the meaning of the original sentence. (C) is idiomatically incorrect; someone is prevented from doing something. (D) is grammatically incorrect. Category: Transitions Difficulty: Medium Issues: reading comprehension Strategic Advice: The first sentence of a paragraph should introduce a new idea but should also place the new paragraph in the context of the passage. Getting to the Answer: The second paragraph ends with information about Morgans invention of the gas mask, and the third begins with information about Morgans other important invention. (E) effectively connects the two ideas. (A) creates an inappropriate contrast between this paragraph and the one before. (B) changes the meaning of the sentence. (C) fails to tie the new paragraph logically to what came before it. (D) is illogical in context, since the author indicates that he remembers his classmates presentation vividly. Category: Reading Comprehension Difficulty: Medium Issues: wordiness Strategic Advice: Read question stems carefully. This one is asking for an answer that would NOT improve the sentence, which means the other four choices must be possible improvements. Getting to the Answer: Of the revisions offered, the only one that wouldnt improve the sentence is (D). (A), (C), and (E) eliminate some of the sentences wordiness. Changing the dash to a semicolon, as (B) suggests, would make the sentence easier to follow.

SAT

Practice Test 2
51

34.

35.

section seven

Section 8 (Math)
1. B Category: Equations Difficulty: Low Strategic Advice: When an SAT question asks for an expression, its usually faster to solve for it directly than to find the parts individually. Getting to the Answer: This question asks you to find the value of (a + b), not of either a or b. You can consider this an equation with only one unknown, and you dont need to break up the parentheses to get to the answer. 25 5(a + b) = 20 25 = 20 + 5(a + b) 25 20 = 5(a + b) 5 = 5(a + b) 1 = (a + b) Category: Ratios Difficulty: Low distance Strategic Advice: The equation rate = can also be written time distance = rate time. Be prepared to manipulate formulas to fit the information given in a specific question. Getting to the Answer: Toms average speed, s, is the rate of travel in miles per hour. The time he spent traveling is t hours. The distance he went is d = st miles. 3. D Category: Angles Difficulty: Low Strategic Advice: Be careful not to eyeball a figure when it is not drawn to scale. Use the information given in the question to get to the answer. Getting to the Answer: The angles x, y, and z make up one side of a straight line, so they sum to 180. You know that x = 35 and y = 70, so you can set up an equation to find z. x + y + z = 180 35 + 70 + z = 180 105 + z = 180 z = 180 105 z = 75 Category: Data Analysis Difficulty: Medium Strategic Advice: Start with the most common Roman numeral and evaluate that statement first. Eliminate answer choices as you go. actual change Getting to the Answer: Use the percent change formula, 100%, original amount to find the increase in the number of colleges in each region. Statement II appears in four of the answer choices, so evaluate it first. According to the graph, there were 10 Southern colleges in 1950 and 50 in 2000. 50 10 40 100% = 100% = 4 100% = 400% 10 10

section eight
2.

4.

SAT

Practice Test 2
52

There was a 400% increase in the number of colleges in the South, so eliminate (C), which does not include II. Statement III: 25 5 20 100% = 100% = 4 100% = 400% 5 5 Now you can eliminate (A) and (B), since they dont include III. Statement I: 50 20 30 3 100% = 100% = 100% = 150% 20 20 2 There was not a 400% increase in the number of colleges in the Northeast, only in the Midwest and the South, so (D) is correct. 5. C Category: Data Analysis Difficulty: Low Strategic Advice: Read the chart carefully. Its easy to make an arithmetic error or misread a number. Getting to the Answer: You can either find the difference in the number of colleges for each region, then add them together, or find the total number of colleges in 1950 and subtract that from the total number of colleges in 2000. Northeast: 50 20 = 30 South: 50 10 = 40 Midwest: 25 5 = 20 West: 60 30 = 30 Total difference: 30 + 40 + 20 + 30 = 120 Category: Polygons Difficulty: Medium Strategic Advice: If you dont remember how to find the area of a parallelogram, you can break the figure into smaller geometric shapes and find their areas separately. Then add the areas together to find the total area of the larger figure. Getting to the Answer: This problem requires you to recognize that, in a parallelogram, the parallel sides are the same length. Therefore, AC and DF both equal 20, and AD and CF both equal 13. The base of triangle ADE is 5 and the hypotenuse, AD, is 13. You should recognize triangle ADE as a 5 : 12 : 13 Pythagorean Triplet, in which AE is 12. If you remember that the area of a parallelogram is base times height, you can find the area now: 20 12 = 240. If you dont recall how to find the area of a parallelogram, consider the triangles ADE and FCB and the rectangle ABFE. Since triangle FCB has the same side lengths and angles as ADE, FCB is also a 5 : 12 : 13 Pythagorean Triplet. Thus, AE and BF both equal 12. Now that all the lengths of the line segments are known, you can determine the area of the two triangles and of the rectangle and add them together to determine the total area of the parallelogram. Area of one triangle: A= 1 bh 2 1 A = (5)(12) 2 A = 30

SAT

Practice Test 2
53

6.

section eight

section eight

Area of rectangle: A = wl A = (12)(15) A = 180 Area of rectangle + area of both triangles = total area of parallelogram 180 + 2(30) = 180 + 60 = 240 7. D Category: Equations Difficulty: Medium Strategic Advice: Draw a figure to keep track of the numbers given in the question stem. Getting to the Answer:

Town A

1 4

400 miles Town B Town C

3 4

Town D

The 400 miles between Town B and Town C is the distance from distance to

1 of the total 4

3 3 1 1 of the total distance. Thats = the total distance. Say x is 4 4 4 2

the total distance traveled, set up an equation, and solve for x. 400 = 1 x 2

400(2) = x 800 = x The total distance traveled is 800 miles. 8. B Category: Coordinate Planes Difficulty: High Strategic Advice: Sketching a graph will often let you eliminate some wrong answer choices on Coordinate Planes questions. Getting to the Answer:
y H(0, 3) 1 K(2, 1) 1 J(4, 1) x

SAT

The center of the circle will be the same distance from the points H, J, and K, since all three points are on the circumference of the circle. A quick sketch should let you eliminate (A) and (E) without too much difficulty. (A) is clearly too far to the left and (E) is much too far up. You can then Backsolve using the distance formula, d = (x1 x2)2 + (y1 y2)2, with the remaining answer choices. The distance between (C) and H is The distance between (C) and J is (0 2)2 + (3 0)2 = (4 2)2 + (1 0)2 = 4+9= 4+1= 13. 5.

Practice Test 2
54

Since two points on the circumference of the circle are different distances from (C), (C) cannot be the center. The distance between (D) and H is The distance between (D) and J is (0 1)2 + (3 1)2 = (4 1)2 + (1 1)2 = 1+4= 9 = 3. 5.

SAT

Practice Test 2
55

Again, the two points are different distances from (D), so you can eliminate (D). The correct answer choice is therefore (B). The distance between (B) and H is The distance between (B) and J is The distance between (B) and K is (0 1)2 + (3 0)2 = (4 1)2 + (1 0)2 = (2 1)2 + (1 0)2 = 1+9= 9+1= 9+1= 10. 10. 10.

Since all three points are the same distance from (B), (B) must be the center of the circle. 9. B Category: Equations Difficulty: Medium Strategic Advice: Use the answer choices to your advantage; do not do more math than necessary. Getting to the Answer: Each of the m buses holds 20 students, so the total number of students in all of the buses is 20m. To determine the number of students that will not fit on the bus, you must subtract the total number of students held by all of the buses, 20m, from the total number of students: 128 20m. Category: Equations Difficulty: Medium Strategic Advice: Many algebra problems require thinking flexibly about expressions. Getting to the Answer: To solve the problem algebraically, rewrite the expression 8m 8n mn in terms of . 9p + 9q p+q 8m 8n 8(m n) 8 m n 8 3 24 2 = = = = = 9p + 9q 9(p + q) 9 p+q 9 4 36 3 Or, Pick Numbers. Let m = 4, n = 1, p = 2, and q = 2. Then given, and 11. A 8m 8n 8(4) 8(1) 32 8 24 2 = = = = . 9p + 9q 9(2) + 9(2) 18 + 18 36 3 mn 41 3 = = , as p+q 2+2 4

10.

Category: Complex Figures Difficulty: Medium Strategic Advice: Whenever you see multiple figures, youll need to move information from one figure to the next. Getting to the Answer: Find the area of the square and the area of the circle, then subtract the area of the circle from the area of the square. Notice that the diameter of the circle is equal to one side of the square, so the length of each side of the square is 2 4 = 8 inches. Area of the circle: A = r 2 A = (4)2 A = 16 in2

section eight

section eight

Area of the square: A = s2 A = 82 A = 64 in2 Area of the shaded region: 64 16 in2 12. E Category: Divisibility Difficulty: Medium Strategic Advice: Picking Numbers is a good strategy for remainder problems. Getting to the Answer: The most manageable number to pick in a remainder problem is the divisor added to the remainder. In this case, thats 7 + 3 = 10. If j = 10, 6j = 60. When 60 is divided by 7, you get 8 with a remainder of 4. Category: Exponents Difficulty: Medium Strategic Advice: Remember the rules for multiplying and dividing exponents. When you multiply the same base with different exponents, keep the base and add the exponents. When you divide, subtract the exponents. Getting to the Answer: (x 4)(x 7) (x 6) 14. E (x8)(x2) x11 = x6 5) (x x10 = (x 5)(x 5) = x10 x5

13.

Category: Permutations and Combinations Difficulty: Medium Strategic Advice: Remember the difference between ordered groups (permutations) and unordered groups (combinations). Getting to the Answer: Audrey is choosing 3 games from 5. Since it doesnt matter what order the games are in, be careful not to count any groups twice. If the games are A, B, C, D, and E, Audrey could select ABC, ABD, ABE, ACD, ACE, ADE, BCD, BCE, BDE, or CDE. Thats 10 possible combinations of three different games. Category: Logic Difficulty: High Strategic Advice: Work methodically to get to the answer. Getting to the Answer: The bowl originally had 8 pieces of fruit. After two children drew bananas, there were no bananas left in the bowl. Therefore, 2 of the original pieces of fruit were bananas. The bowl originally contained 2 bananas and 6 apples and oranges. The only combination of apples and oranges that meets the restriction of twice as many apples as oranges is 4 apples and 2 oranges. The children drew one of each, so the bowl is left with 3 apples and 1 orange. Category: Triangles Difficulty: Medium Strategic Advice: Always be on the lookout for special and similar triangles. Getting to the Answer: Since DB and EC are perpendicular to AC, triangles ABD and ACE are both right triangles. You also know that triangle ABD and triangle ACE share the angle at point A. Therefore, the third angle in each triangle will also be the same. The two triangles are similar. They have the same angles, and the lengths of the sides of the larger triangle are a constant multiple of the lengths of the sides of the smaller triangle. You know that AB is 4 and the corresponding side

15.

16.

SAT

Practice Test 2
56

SAT length of the larger triangle, AC, is 8. You also know that DB is 3 and the Practice Test 2 corresponding side length of the larger triangle is EC. Use these lengths to set up a 57 proportion to solve for EC.

3 4 = 8 EC 4EC = 3(8) 4EC = 24 EC = 6

section eight

Section 9 (Critical Reading)


1. B Category: Definition Difficulty: Low Strategic Advice: On two-blank Sentence Completions, start with the blank that seems easier to predict. Getting to the Answer: The second half of the sentence describes the dcor at the Plaza Hotel, which you might describe as lush or luxurious, both great predictions for the first blank. How would the small-town girls react in such an environment? Predict that they were impressed or overwhelmed. (A) Nothing in the sentence indicates a somber atmosphere in the hotel, and amused inappropriately suggests that the hotel is funny to the girls. (C) The hotel may be novel, but such amazing decorations would not make the girls jaded, or cynical. (D) The girls could be considered astounded, but the Plaza Hotel, with all of its fancy decorations, is anything but mundane, or boring. (E) There are no Keywords to denote the hotel as joyous; stymied makes no sense in context. somber: gloomy; melancholy lavish: extravagant; immoderate novel: unusual; markedly different jaded: worn out; sated mundane: typical; commonplace stymied: thwarted; stumped Category: Contrast Difficulty: Medium Strategic Advice: Pay attention to Keywords with two-blank sentences. Do they imply a contrast between the missing words or similar meanings/charges? Getting to the Answer: The first blank talks about the success of the system of checks and balances, which must mean America is less susceptible to difficulties. However signals contrast, so predict that the second missing word will have a negative charge. (A) Filling the governmental offices with qualification does not make sense. (B) This could be tempting, but it is incorrect; a system of checks and balances should not make America less averse, or reluctant, to corruption; the system failing should not fill the governmental offices with encumbrance, or burden. (D) No country would be dependent on corruption; you cannot fill something with impossibility. (E) To be ignorant of corruption would mean the system is not working; filling with oversight does not make sense in context. averse: opposed; strongly disinclined encumbrance: a burden or impediment Category: Definition Difficulty: Medium Strategic Advice: Use Keywords to your advantage as you make your prediction. Getting to the Answer: But signals a contrast between the way Emily truly feels and the way her confidence and insistence upon proper manners come off to her classmates. Since she doesnt think shes better than anyone else, predict the

section nine
2.

3.

SAT

Practice Test 2
58

SAT opposite for the blank, which describes her classmates misunderstanding. Practice Test 2 (A) Confidence and insistence on proper manners wouldnt make someone come 59 off as deceptive. (B) Emilys proper manners may make her seem attentive to details about her own behavior, but it matches awkwardly with confidence and doesnt provide the necessary contrast. (D) Disagreeable does not relate to confidence and proper manners. (E) Someone with proper manners may seem agreeable, but again, this does not provide the necessary contrast. deceptive: misleading pretentious: marked by an extravagant outward show; ostentatious

4.

Category: Contrast Difficulty: High Strategic Advice: Be on the lookout for clues provided by both Keywords and sentence structure. What is the logical relationship of the blanks to each other? Getting to the Answer: The big clue here is even after, showing that Neal remains optimistic despite news of rejection. The second blank picks up on this idea, so predict that Neal is still certain of success. (A) With Neals tenacity, the situation for him wouldnt be bleak. (B) Becoming a doctor is Neals dream, so he should not feel dissatisfied about it; achieve talent is awkward. (C) Neal can feel hopeful about his dream, but achieving humility doesnt make sense in context. (E) Neal would not be submissive towards his dream and to achieve ignorance wouldnt be a goal of his. humility: a disposition to be humble impervious: incapable of being penetrated or affected Category: Definition Difficulty: Medium Strategic Advice: Remember that a familiar word might derive from an older word with a related meaning. If you dont see your prediction among the choices, examine roots, think of familiar contexts, and remember your elimination strategies. Getting to the Answer: The first part of the sentence describes different types of roses, while the second goes on to say that the red rose is the most cherished member of some larger group. Predict a word that describes flowers in general. (B) A sierra is a range of mountains with an uneven border. (C) An archipelago is a large group of islands. (D) A flotilla is a fleet of small ships. (E) A savanna is flat, treeless grassland within tropical regions. Category: Contrast Difficulty: Low Strategic Advice: Attention to Keywords in the sentence will lead you to a strong prediction. Getting to the Answer: You know that the young man is unsure of his true talent. You can therefore predict that his approach to the microphone was modest or timid. (A) This is the opposite of your prediction, since a young man unsure of his talent wouldnt approach the microphone with confidence. (B) Again, if the young man was unsure, he wouldnt approach the microphone

5.

6.

section nine

section nine

quickly. (D) This is another opposite, since fear or insecurity wouldnt normally inspire a brazen action. (E) This makes for an awkward construction when read back into the sentence, since insecurity and faithfulness dont have a particularly strong connection. brazen: recklessly bold; insolent

Tourism in Mexico
7. E Category: Global Difficulty: Medium Strategic Advice: Global questions deal with the passage as a whole; the correct answer choice will summarize main points rather than refer to specific examples. Getting to the Answer: The author is critical of tourist-oriented beach towns, using such words as artificial, so the overall point is that the towns do not reflect the culture of the area. Look for an answer choice that makes a similar claim. (A) Opposite; the author believes that these towns are not representative of their surroundings. (B) Distortion; this relates closest to the gym, which is said to be easy to navigate. (C) Misused Detail; the areas lack sufficient resources, and paragraph 3 contradicts the notion that the towns are not equipped to accommodate visitors. (D) Distortion; while the author suggests that the beach towns wont teach the traveler about the area, calling them reliant on the ignorance of travelers is far too negative and misrepresents the authors points. Category: Detail Difficulty: Medium Strategic Advice: A Detail question might use general language for answer choices. Think about how you might describe the relevant details in less specific terms. Getting to the Answer: The author makes only one direct general statement about specialized areas: that any living thing must itself be specialized or acclimate quickly. (A) Out of Scope; the author makes no comment on the uniqueness of specialized areas in general. (B) Distortion; forbidding refers to an example rather than a general statement, and unpleasant expresses a judgment the author does not make. (D) Misused Detail; this refers to the gymnasium comparison later in the passage. (E) Distortion; the specialized part of the area refers to environment generally, while this answer choice is specifically talking about the beach towns. Category: Vocab-in-Context Difficulty: Low Strategic Advice: Vocab-in-Context questions can be treated like Sentence Completions. Look for an answer choice that could replace the given word, making sure that it matches the meaning in this particular context. Getting to the Answer: The author says that beach towns do not characterize the history and culture as well as other areas. The author is referring to an accurate portrayal of the land, so look for an answer choice matching this. (B) Extreme; placing emphasis upon something is doing more than portraying it

8.

9.

SAT

Practice Test 2
60

accurately. (C) Opposite; the beach towns could draw attention away from the history, but characterize is preceded by not in the cited text. (D) Distortion; explain makes the other portions of the country far too active and results in awkward phrasing. (E) Opposite; the beach towns would likely distort the history, but again, characterize is preceded with not in the cited text. 10. A Category: Inference Difficulty: Medium Strategic Advice: Inference questions require following the logic of the passage; the correct answer can be deduced from the text. Getting to the Answer: The author brings up the fact that the beach towns must import resources to accommodate a great number of people, while the altiplano has these resources naturally. If one considers the beach towns as representative of all of Mexico, one might therefore incorrectly assume that the country is dependent on outside resources. (B) Out of Scope; the author states that there is a great number of people being accommodated but makes no comparison to other parts of the country. (C) Opposite; the author implies the beach-going tourist would see the country as being less diverse than it actually is. (D) Opposite; the altiplano is temperate, while the beach is tropical. (E) Opposite; the coast depends on other areas for resources, not the other way around. Category: Function Difficulty: Medium Strategic Advice: Function questions like this one are based on the authors overall purpose. The correct answer choice will contribute to the logic of the argument as a whole. Getting to the Answer: The author starts the introduction of the Mayans with While the tourist, which suggests a comparison. The selection describes the Mayanss means of survival in contrast to what tourists find desirable. You can predict that the subject of the Mayans is used to illustrate the differences between what they looked for on the coast (water) with what tourists look for (beauty, tropical weather). (A) Distortion; the Mayans lived on the coast, not the altiplano. (C) Opposite; the passage describes the difficulty of settling in tropical environments. (D) Distortion; current locals are not mentioned in the selection. (E) Distortion; the author is contrasting the experiences of the two groups, not taking tourists to task. Category: Inference Difficulty: Medium Strategic Advice: Inference questions like this one rely on keeping the authors opinions straight; the correct answer will be the only one that falls in line with the authors overall points and underlying logic. Getting to the Answer: The paragraph about the gymnasium lists ways in which its function is similar to that of a beach town. The correct answer choice will restate one of the points the author makes, while keeping its logic intact, such as a design

SAT

Practice Test 2
61

11.

12.

section nine

section nine

based on the visitors demands and desires. (A) Opposite; the gymnasium is said to provide patrons with a familiar experience. (B) Opposite; both are intended for short-term stays. (C) Out of Scope; the environment within a gymnasium is not mentioned in the passage. (E) Misused Detail; this refers to the markets, not the gyms. 13. C Category: Function Difficulty: High Strategic Advice: Function questions are based on the authors overall argument; the correct answer will contribute to it. Getting to the Answer: The author completes the sentence by saying that a customer in a gymnasium would find similar equipment and have a similar experience due to familiarity with this basic range of functions. The author is making a point that gyms as a whole are put together in a specific way that allows for a repeatable experience. Look for an answer choice that follows this line of logic. (A) Distortion; the selection focuses on the planning behind gyms, not what is accomplished. (B) Extreme; the author is not making a favorable comparison with regard to beach towns, but this paragraph does not criticize gyms. (D) Misused Detail; the author says that varying from the model would be a disservice. (E) Distortion; basic refers to design, not use. Category: Inference Difficulty: Low Strategic Advice: Inference questions like this one focus on deducing the authors view and applying it; the correct answer will stay close to the text and draw a strong conclusion from it. Getting to the Answer: The paragraph focuses primarily on describing the way the market is influenced by its surroundings, which the author applies to the altiplano as well. (A) Out of Scope; the author does not mention the aesthetic values of the market. (C) Out of Scope; the tourist presence at local markets is not mentioned. (D) Misused Detail; this refers to the apple/guava example, which is not used to make a point about limits in produce. (E) Out of Scope; one tourist story is mentioned, but the number of tourists is never mentioned. Category: Detail Difficulty: Medium Strategic Advice: Detail questions rely directly on the text itself; the correct answer will restate specific facts from the passage and may use more general language to do so. Getting to the Answer: The preceding sentence talks about the travelers wishes going unfulfilled, resulting in authentic experiences of local culture. Predict that the correct answer will describe this situation. (A) Extreme; the narrator does not categorize this as a negative event. (C) Distortion; the traveler may be looking for something different, but nothing is

14.

15.

SAT

Practice Test 2
62

categorized as unsuitable. (D) Distortion; this is not presented as occurring in coastal Mexico. (E) Out of Scope; the authors preferences are not addressed. 16. E Category: Inference Difficulty: Medium Strategic Advice: When Inference questions cite specific text, pay special attention to the way this text relates to the logic of the authors argument as a whole. Getting to the Answer: The paragraph concludes that the best way to travel is to experience unfamiliar areas, implying that this is what the traveler wants from a trip in the first place. Look for an answer choice that reflects this. (A) Opposite; the author believes the traveler would enjoy seeing foreign sights. (B) Misused Detail; the author is not endorsing destinations based on climate. (C) Out of Scope; neither the languages spoken in different areas nor the tourists knowledge of them are ever addressed in the passage. (D) Out of Scope; the passage never brings up traveling companions or lack thereof. Category: Inference Difficulty: High Strategic Advice: Inference questions like this require defining a term the author uses from the authors own particular perspective. Getting to the Answer: The author sees destinations that best reflect local culture as being the best places to go. Look for an answer choice that reflects an emphasis on an appreciation of culture. (A) Out of Scope; the author doesnt describe one destination as more comfortable than another. (B) Out of Scope; the author never compares prices. (D) Distortion; the author doesnt make any distinctions regarding the pleasantness of the country. (E) Distortion; if anything, the author mentions more about the scenic beauty of the coast. Category: Global Difficulty: Low Strategic Advice: Global questions deal with the passage as a whole; the correct answer will paraphrase the authors main point. Getting to the Answer: The last paragraph recommends traveling to destinations defined by local culture rather than those equipped for tourism. Look for an answer choice that reflects this. (A) Distortion; the authors point is general and comes from comparison rather than detail. (B) Out of Scope; the author focuses on two specific areas, not the country as a whole. (C) Distortion; the author never attempts to warn the reader. (E) Extreme; while the author certainly prefers areas outside of beach towns, the idea that beach towns distort local history goes too far and is not the point the author intends to make. Category: Global Difficulty: Medium

SAT

Practice Test 2
63

17.

18.

19.

section nine

Strategic Advice: Global questions deal with the passage as a whole, and the correct answer choice will reflect this. Getting to the Answer: The passage compares tourist-oriented areas with those that are influenced by their surroundings, with preference given to the latter. (A) Out of Scope; this is not a major part of the passage and the author considers it relatively unimportant. (B) Misused Detail; this refers to the description of the altiplano, but it is not a main idea. (C) Extreme; the passage is about both where the author feels it is culturally worthwhile to visit and where it is not; this is too one-sided. (E) Out of Scope; this is too specific, and unique is not the authors concern; authentic is more like it.

SAT

Practice Test 2
64

section nine

Section 10 (Writing)
1. D Category: Passives Difficulty: Medium Issues: style problems, wordiness Strategic Advice: The passive voice will not always be incorrect on the SAT, but check for an answer choice that makes the sentence active. Getting to the Answer: The underlined phrase is written in the passive voice, which also makes it unnecessarily wordy. (D) is both active and concise. (B) changes the meaning of the sentence, making Patricia the one being noticed, rather than the one doing the noticing. (C) and (E) are unnecessarily wordy. Category: Correct as Written Difficulty: Medium Issues: verb tenses, wordiness Strategic Advice: Expect between five and eight Writing section questions to be correct as written. Getting to the Answer: The sentence contains no error. (B) introduces an inconsistent verb tense. (C), (D), and (E) are all unnecessarily wordy. Category: Run-ons and Fragments Difficulty: Medium Issues: passives, pronouns Strategic Advice: There are a number of ways to fix a run-on sentence, but only one answer choice will correct the problem without introducing additional errors. Getting to the Answer: As written, the sentence incorrectly joins two independent clauses with a comma. (C) corrects the error by making the second clause dependent. (B) uses the passive voice unnecessarily and uses the inappropriate pronoun which to refer to people. (D) creates a sentence fragment. (E) changes the meaning of the sentence; the jury members, not the decisions, are chosen by the attorneys. Category: Correct as Written Difficulty: Medium Issues: wordiness, parallelism Strategic Advice: Eliminate wrong answer choices methodically; if you dont spot an error, choose (A). Getting to the Answer: This sentence is correct as written. (B), (C), and (E) are unnecessarily wordy. (D) violates the rules of parallel structure. Category: SubjectVerb/Wordiness Difficulty: High Issues: idioms Strategic Advice: A verb must agree with its subject noun, which may not be the noun closest to it in the sentence. Getting to the Answer: Although the singular noun mouthwash appears closer to

SAT

Practice Test 2
65

2.

3.

4.

5.

section ten

it in the sentence, the subject of the singular verb hasresonated is actually the plural directives. Both (D) and (E) correct the agreement error, but for changing in (E) is idiomatically incorrect in context. (B) does not address either error. (C) does not address the subjectverb error.

section ten

6.

Category: Parallelism Difficulty: Medium Issues: none Strategic Advice: Items in a series or list must be parallel in form. Getting to the Answer: The third item in this seriesit will calldoes not parallel the first two, sends and communicates. (E) makes the third item parallel. None of the other choices creates parallel structure. Category: Wordiness Difficulty: High Issues: transitions Strategic Advice: If you dont spot an error in grammar or usage, check for errors in style. Getting to the Answer: This sentence is unnecessarily wordy. (B) makes the sentence more concise without changing its meaning. (C), (D), and (E) are still unnecessarily wordy; additionally, (D) creates an illogical relationship between the two clauses. Category: Idioms Difficulty: Medium Issues: wordiness Strategic Advice: Prepositions are the most frequently tested problem in Idiom questions. Getting to the Answer: About their talking is idiomatically incorrect with to stop; from is the appropriate preposition in this context. Both (B) and (C) correct the error, but (B) is unnecessarily wordy. (D) and (E) are both idiomatically incorrect. Category: Modifiers Difficulty: High Issues: none Strategic Advice: If a sentence does not make clear what a modifying word or phrase is referring to, look for an answer choice that does. Getting to the Answer: Here, its unclear which group of players actually uses the aluminum bats. (B) clarifies the modifier. (C), (D), and (E) do not address the error. Category: Wordiness Difficulty: Low Issues: style problems Strategic Advice: Long answer choices will not always be incorrect, but if an idea can be expressed more concisely, the correct answer choice will do so.

7.

8.

9.

10.

SAT

Practice Test 2
66

SAT Getting to the Answer: While grammatically correct, this sentence is unnecessarily Practice Test 2 wordy. (E) says the same thing in a more concise way. 67 (B) and (D) are even wordier than the original. (C) leaves the meaning of the second clause incomplete.

11.

Category: Correct as Written Difficulty: Medium Issues: wordiness, passives Strategic Advice: Never discount the possibility that a sentence is correct as written. Getting to the Answer: This sentence is correct as written. (B) and (C) are unnecessarily wordy. (D) creates a run-on sentence. (E) introduces the passive voice unnecessarily. Category: Parallelism Difficulty: Medium Issues: none Strategic Advice: Items serving parallel functions must have parallel forms. Getting to the Answer: The underlined selection should parallel the first two items in the list: style and way. The only answer choice that does so is (C). Category: Run-ons and Fragments Difficulty: Medium Issues: style problems, transitions, wordiness Strategic Advice: Two independent clauses cannot be joined by just a comma. Getting to the Answer: As written, this sentence is a run-on. (B) corrects the error by making the second clause subordinate. (C) misuses a semicolon; when combining clauses with a FANBOYS conjunction, a comma is used between them. (D) does not address the run-on error and reverses the cause-and-effect relationship between the clauses. (E) is unnecessarily wordy. Category: Modifiers Difficulty: High Issues: wordiness, passives, ambiguity Strategic Advice: A modifying word or phrase generally modifies the first noun or pronoun that follows it. Getting to the Answer: Because of the way this sentence is structured, they, rather than The professor, are the ones who reviewed the test results. (D) fixes this by restructuring the sentence. (B) doesnt address the error. (C) and (E) are overly wordy; additionally, (C) introduces the passive voice unnecessarily, and (E) uses the pronoun it without a clear antecedent.

12.

13.

14.

section ten

Das könnte Ihnen auch gefallen